HU NSEN 3 STR O N G · Five Hundred Mathematical Challenges,Complex Numbers From A to … Z ,...

159
eroberogeday lwm pl:ún briBaØabR&tKNitviTüa nig BaNiC¢kmμ H U N SE N × S T R O N G 3 Problems and Solutions

Transcript of HU NSEN 3 STR O N G · Five Hundred Mathematical Challenges,Complex Numbers From A to … Z ,...

Page 1: HU NSEN 3 STR O N G · Five Hundred Mathematical Challenges,Complex Numbers From A to … Z , Mathematical Olympiad in China,Mathematical Olympiad Challenges, Mathematical Olympiad

eroberogeday lwm pl:ún briBaØabR&tKNitviTüa nig BaNiC¢kmμ

H U N S E N×

S T R O N G

3

Problems and Solutions

Page 2: HU NSEN 3 STR O N G · Five Hundred Mathematical Challenges,Complex Numbers From A to … Z , Mathematical Olympiad in China,Mathematical Olympiad Challenges, Mathematical Olympiad
Page 3: HU NSEN 3 STR O N G · Five Hundred Mathematical Challenges,Complex Numbers From A to … Z , Mathematical Olympiad in China,Mathematical Olympiad Challenges, Mathematical Olympiad

GñkcUlrYmRtYtBinitübec©keTs

elak lwm qun elak Esn Bisidæ elakRs I Tuy rINa elak Titü emg elak RBwm sunitü elak pl b‘unqay

GñkRtYtBinitüGkçraviruTæ elak lwm miKþsir

karIkMuBüÚT&r kBaØa lI KuNÑaka

GñkniBnæ nig eroberog elak lwm plþún nig elak Esn Bisidæ

Page 4: HU NSEN 3 STR O N G · Five Hundred Mathematical Challenges,Complex Numbers From A to … Z , Mathematical Olympiad in China,Mathematical Olympiad Challenges, Mathematical Olympiad

GarmÖkfa

esovePA KNitviTüaCMuvijBiPBelak EdlGñksikßakMBugkan´ enAkñúgéden£ xMJúáTáneroberogeLIgkñúgeKalbMngTukCaÉksar sRmab´ CaCMnYydl´GñksikßaykeTAsikßaRsavRCavedayxøÜnÉg nig müageTot kñúgeKalbMngcUlrYmelIksÞÜyvis&yKNitviTüaenARbeTskm<úCaeyIg [kan´EtrIkceRmInEfmeTotedIm,Ibeg;InFnFanmnusß[mankanEteRcIn edIm,ICYyGPivDÄn_RbeTsCatirbseyIg . enAkñúgesoven£eyIgxMJúánxitxMRsavRCaveRCIserIsyklMhat´ya¨g sRmaMgbMputBIÉksarbreTsyageRcInbMputdUcCa 103 Trigonometry Problems ,

Five Hundred Mathematical Challenges,Complex Numbers From A to … Z , Mathematical Olympiad in China,Mathematical Olympiad Challenges, Mathematical Olympiad Treasures, International Mathematical Olympiads 1959-1977,

The IMO Compendium ( 1959-2004 ) …nigÉksarbreTsepßg@eTottamy¼ Internet ykmkeFIVdMeNa£Rsayya¨gek,a£k,ayEdlGac[elakGñk

gayyl´qab´cgcaMGMBIsil,£énkareda£RsayTaMgGs´en£ . b¨uEnþeTa£Ca ya¨gNak¾eday kgV£xat nig kMhusqþgedayGectnaRákdCamanTaMg bec©keTs nig GkçraviruTæ . GaRs&yehtuen£ eyIgxMJúCaGñkeroberogrgcaMedayrIkrayCanic©nUv mtiri£Kn´EbbsSabnaBIsMNakGñksikßakñúgRKbmCÄdæanedIm,ICYyEklMG esovePAen£[ánkan´EtsuRkiRtPaBEfmeTot .

Page 5: HU NSEN 3 STR O N G · Five Hundred Mathematical Challenges,Complex Numbers From A to … Z , Mathematical Olympiad in China,Mathematical Olympiad Challenges, Mathematical Olympiad

CaTIbBa©b´en£eyIgxMJúGñkeroberogsUmeKarBCUnBrdlGñksikßaTaMgGs´ [mansuxPaBmaMmYn nig TTYlC&yCMn£RKb´Parkic© .

átdMbgéf¶TI07 mkra 2009 GñkniBnæ lwm plþún Tel : 017 768 246

Page 6: HU NSEN 3 STR O N G · Five Hundred Mathematical Challenges,Complex Numbers From A to … Z , Mathematical Olympiad in China,Mathematical Olympiad Challenges, Mathematical Olympiad

lwm plþún nig Esn Bisidæ

H U N S E N× 3

S T R O N G

Problems and Solutions

Page 7: HU NSEN 3 STR O N G · Five Hundred Mathematical Challenges,Complex Numbers From A to … Z , Mathematical Olympiad in China,Mathematical Olympiad Challenges, Mathematical Olympiad

rkßasiT§iRKb´yag

Page 8: HU NSEN 3 STR O N G · Five Hundred Mathematical Challenges,Complex Numbers From A to … Z , Mathematical Olympiad in China,Mathematical Olympiad Challenges, Mathematical Olympiad

KNitviTüaCMuvijBiPBelak

EpñkRbFanlMhat

- 1 - PaK1

Page 9: HU NSEN 3 STR O N G · Five Hundred Mathematical Challenges,Complex Numbers From A to … Z , Mathematical Olympiad in China,Mathematical Olympiad Challenges, Mathematical Olympiad

KNitviTüaCMuvijBiPBelak

- 2 - PaK1

EpñkRbFanlMhat´ 1-BIcMnYnBitviC¢man nig x y

epÞógpÞat´TMnak´TMng 11y3x4 =+ cUrkMntrtémøGtibrmaénGnuKmn_ ½

6 )3)(7y)(x()y,x(f y2x +++= 2 enøa¼-cUrkMntRKb´témø kñúgc x [

2;0] π edaydwgfa ½

2xcos13

xsin13 ++

− 4=

mYy3-enAkñúgRtIekaN Edl cAB,bAC,aBCABC === cUrbgHajfa

cba

2Asin

+≤ rYcsresrTMnak´TMngBIreTotEdl

tag[ce RsedogKña . 4-eKyk I nøa¼ ]

4;

4[ ππ− . cUrkMnt´GnuKmn_

os(s

f

kMntelI [ − ebIeKdwgfa cxsin)x2in]1,1 xf += 2 cMeBa¼RKb´ kñúgcenøa¼ .

jfa t

rYcsRmYl f )x(tan x I

5-cUrbgHa tanatana2tana3an atana2tana3=−− Edl

2ka π

≠ RKb´cMnYnKtrWTLaTIhV . k

Page 10: HU NSEN 3 STR O N G · Five Hundred Mathematical Challenges,Complex Numbers From A to … Z , Mathematical Olympiad in China,Mathematical Olympiad Challenges, Mathematical Olympiad

KNitviTüaCMuvijBiPBelak

- 3 - PaK1

6-cUrbgHajfa ( )2

ab21xcos

b1xsin

a1 +≥⎟⎠⎞

⎜⎝⎛ +⎟

⎠⎞

⎜⎝⎛ +

cMeBa¼RKb´ 2

xb,0a 0,0 π<>> <

-e ue mYyEdl 7 K[ct kaNEkg ABCD AB3BC = . aBIrcMnucénRCug Edl QB = P nig Q C PP]BC[ QC= .

cUrbgHajfa DPDBCDQC C∠+∠=∠ . CacMnYnkMupøicBIr . 8-eK[ 1z nig 2z

cUrRsayfa ( )22

21

221

22 2|zz||zz| 1 |z||z| +=−++

9-eK[ 2 Ca nig cMnYnkMupøicBIrEdl 1z z 1|z||z| 21 == . nig z.z 21 −≠ 1

cUrRsayfa 21

21

zz1+ CacMnYnBzz + itmYy .

10-KNnalImItxageRkam ½

4x

4x1212...121212lim

4x −

−++++++ man n ¦skaer .

Page 11: HU NSEN 3 STR O N G · Five Hundred Mathematical Challenges,Complex Numbers From A to … Z , Mathematical Olympiad in China,Mathematical Olympiad Challenges, Mathematical Olympiad

KNitviTüaCMuvijBiPBelak

- 4 - PaK1

11-eK[sIVúténcMnYnBit kMnt´eday ½

=

)u( n

0 nig 1u1u4 n +u6u4

uu 2n

3n

4n

1n ++=+ RKb´

INn∈

cUrbgHajfa4

n1n u1

u ⎟⎟⎠

⎜⎜⎝

+=++

111⎞⎛ RKb´ INn∈

CaGnuKmn_én . KNna

1)(1n2 +

+−

. nmMu kat g´cMnuc .

rYcKNna nu n

12- x1).....(xxxx1(P

n2422n −++−= )x

13-eK[RtIekaN ABC mYymanRCugRbEvg c,ba ,

knø¼bnÞatBu¼é ] RtC AB[ D

cUrRsayfa 2Ccos

baab2CD+

= . KNnaplbUk 14-

1Sn !n.n.....!3.3!2.2!1. ++++ =

Edl n(n!n 1.2.3)....2n)(1 −− . KNnatémø

=

15- )29tan3)...(3tan3)(2tan3)(1tan3(A oooo ++++=

Page 12: HU NSEN 3 STR O N G · Five Hundred Mathematical Challenges,Complex Numbers From A to … Z , Mathematical Olympiad in China,Mathematical Olympiad Challenges, Mathematical Olympiad

KNitviTüaCMuvijBiPBelak

- 5 - PaK1

16- itEdleK[ CacMnYnB epÞógpÞat´lkçx&NÐ ½ z,y,x

ycosxcos 0zcos =++ cos nig x3cos 0z3cosy3 =++ cx2s cUrbgHajfa 2cosy2osco 0z ≤ .

k ug .

cUrRsayfa ½

17-eK[RtIekaN mYy . ABC

bnÞat´Bu¼mMu A at´RCC,B, ]AB[,]AC[,]BC[

erogKñaRtg´ ,'A 'C,'B

0'2

'AA2 =

CC

)BAsin(

'BB

)2

ACsin()CBsin( −

+

+

18-eK[BhuFa n)asasinx()x(P co+= n∈Edl

nwg *IN

cUrrksMNlénviFIEckrvag )x(P 1x2 + .

&n19-eda¼RsayRbB æsmIkar ⎩⎨

− yyx4 ⎧

−=

+=+

baxbayx

4

3

CacMnYnBitEdl Edl a nig b 0ba ≠+ . 0-cUrbgHajfa 2 9

)1nn)(1nn(19999

1n44 =⎟

⎞⎜⎝

⎛++++

∑=

Page 13: HU NSEN 3 STR O N G · Five Hundred Mathematical Challenges,Complex Numbers From A to … Z , Mathematical Olympiad in China,Mathematical Olympiad Challenges, Mathematical Olympiad

KNitviTüaCMuvijBiPBelak

- 6 - PaK1

21- IeK[GnuKmn_ kMnt´RKb´ f }0;1{Rx −−∈ eday ½ 1x)

x1(f)x(f)1x2(x + +=+ .

cUrKNna )2009(f....)3(f)2(f)1(fS ++++= 22- éncMnYnKtcUrkMnt´RKb´KU ´viC¢manebIeKdwgfa ½ )n;m(

)m(13nm 22 n+=+ . 23-eK[RtIekaN C m g´ A .AB YyEkgRt

k nig eKy F]AC[E∈ ]AB[∈ Edl ABCAEF ∠=∠

nig erogKñaCaeCIgcMeNalEkgén nig .

ayfa

nig ACBAF ∠=∠ . E

eKtag E 'F ' E F

elIRCug BC[ ]

cUrRs BC'FFEFE'E ≤++ ?

24-eK[ CabIcMnYnBitviC¢man . cUrRsaybBa¢ak´fa ½ c,b,a

2)bb)cb(4a 33 33 a(4c

ba)ac(4

accb3 33 33

≤++

++

+++

+++

+

it =

25-eK[sIVútcMnYnB−⎩

⎨⎧

≥∀+

==

−+ 1n,2a2a1aa

1n1n

10 an

Page 14: HU NSEN 3 STR O N G · Five Hundred Mathematical Challenges,Complex Numbers From A to … Z , Mathematical Olympiad in China,Mathematical Olympiad Challenges, Mathematical Olympiad

KNitviTüaCMuvijBiPBelak

- 7 - PaK1

cUrRsayfa 1nn aaa +1n 2 +×= .

26-kñúgRtIekaN mYycUrRsaybBa¢akfa ½ ABC

r2Csin

2Bsin

2Asin

R4111≥++

ig Edl r n R C IekaN . CabIcMnYnBitviC¢manEdl akaMrgVg´carwkkñúg nig carwkeRkARt

27-eK[ b,a c, 1cbaabc4 +++= cUrbgHajfa ½

)cabcab(2c

bab

ca

cb≥

+++

+ a 222222

+++ 28-eK kMnt´eday ½

101n

[sIVút }a{ n

1a0 = nig 4na....a.aa +=+ a¼RKb . a

cMeB ´ n ≥ 1

cUrbgHajf 2a 1n =+ cMeBa¼RKb´ n ≥an − . CabIcMnYnBitviC¢manEdl cab

1

29-eK[ ,a bc,b 1ca =++ . cUrbgHajfa 16)

a1c()

c1b()

b1a( 222 ≥+++++

30-eK[ 432 222,

2cos222,

2cos22

2cos2 π

=++π

=+π

= IcUrrkrUbmnþTUeTA nig RsaybBa¢ak´

mnþena¼pg . BI«TahrN_xagelrUb

Page 15: HU NSEN 3 STR O N G · Five Hundred Mathematical Challenges,Complex Numbers From A to … Z , Mathematical Olympiad in China,Mathematical Olympiad Challenges, Mathematical Olympiad

KNitviTüaCMuvijBiPBelak

- 8 - PaK1

31-edayeFIVvicart

dac´nwg Canic©cMeBa¼RKb´ μCati n .

n,7 ∈

amkMeNIncUrRsaybBa¢akfacMnYn ½1n1n6

n 92A ++ += Eck 11

cMnYnKtFm32-eK[ N61009743831E nnnn

n I−+−=

acnwg Canic©RKb´ .

cUrbgHajfa nE Eckd 187 INn∈

33-cUrRsaybBa¢akfa 1n21

1 <+n1.....

31

21

++

+++ viC¢man .

cUrRsaybBa¢akfa ½ 34-eK[ n cMnYnBit n321 a.........;;a;a;a

nn321n321 na..........aaa ≥++++ a......a.a.a

35-eda¼RsayRbB&næsmIkar ½

⎩⎨

tanxtan 33

=+

=++

34ytanytanx28ytanytanxtan6xtan 4224

Edl 2

x0 π< . y <<

´RtIekaNmYynigtag énRtIekaN .

36-eK[ c,b,a CaRCugrbs p

Caknø¼brimaRt

Page 16: HU NSEN 3 STR O N G · Five Hundred Mathematical Challenges,Complex Numbers From A to … Z , Mathematical Olympiad in China,Mathematical Olympiad Challenges, Mathematical Olympiad

KNitviTüaCMuvijBiPBelak

- 9 - PaK1

cUrbgHajfa ⎟⎠⎞

⎜⎝⎛ ++≥

−+

−+

− bp1

ap1

c1

b1

a12

cp1

-k¿cUrRsaybBa¢akfa ½ 37

3n41n4

1n41n4

1n43n4

<+−

<+−

+− cMeBa¼RKb´

x¿TajbgHajfa ½ *INn∈

3n43

)1n4(.......139)1n4(.......117

1n21

+×××−×××

+ 53

+<

××

<

CabIcMnYnBitviC¢man . 38-eK[ ;b;a c

cUrbgHajfa 23

bac

acb

cba

≥+

++

++

39-k¿KNna n)

....2.1

1++ Edl 2n >

1n(1

4.31

3.21

−++

x¿edayeRbIvismPaB GMAM − én n( )1− cMnYnxageRkam ½

n)11;

3.21;

2.11

− cUrbgHajfa nn

n(;

4.31 )!n( 2< .

CabIcMnYnBitxusBIsUnü .

fa

40-eK[ b;a c;

cUrbgHajac

cb

ba

ac

cb 222

++≥⎟⎠⎞

⎜⎝⎛+⎟

⎞⎜⎝⎛+⎟

⎞ ba

⎠⎠⎜⎝⎛

CabIcMnYnBitviC¢manminsUcUrbgHajfa

41-eK[ y;x nü . z;

222222 zyx9

z1

y1

x1

++≥++

Page 17: HU NSEN 3 STR O N G · Five Hundred Mathematical Challenges,Complex Numbers From A to … Z , Mathematical Olympiad in China,Mathematical Olympiad Challenges, Mathematical Olympiad

KNitviTüaCMuvijBiPBelak

- 10 - PaK1

42-eKman xlog11

aay nig −= ylog11

aaz −=

cUrRsayfa zlog11

aax −= .

43-eKBinitüsIVúténcMnYnBit kMnt´eday )u( n⎪⎩

⎪⎨

−−

=

=

+ 1u24u5

u

4u

n

n1n

1

cUrKNna CaGnuKmn_én .

44-eKBinitüsIVúténcMnYnBit kMnt´eday

nu n

)u( n⎪⎩

⎪⎨

−−

=

=

+ 1u4u3

u

3u

n

n1n

1

45-eKBinitüsIVúténcMnYnBit n kMnt´eday ½ cUrKNna nu CaGnuKmn_én . n

u( )

⎩⎨

∈=+ *INn,uu 1n

⎧ = 4u

n

1

_én 46-eK[ Bit kMnt´eday ½ cUrKNna nu CaGnuKmn n .

sIVúténcMnYn )u( n

5u0 = nig 5nu21u nn ++ ( INn1 =+ ∈ )

rKNna énsIVút CaGnuKmn_én . cU nu )u( n n

Page 18: HU NSEN 3 STR O N G · Five Hundred Mathematical Challenges,Complex Numbers From A to … Z , Mathematical Olympiad in China,Mathematical Olympiad Challenges, Mathematical Olympiad

KNitviTüaCMuvijBiPBelak

- 11 - PaK1

47-edayeRbIGnumanrYmKNitviTüacUrKNnalImItxageRkam ½

2x

2x22.....222limL

2xn −−++++

.

8-eK[sIVúténcMnYnBit k t´elI eday ½

++=

( man n ra¨DIkal´ ) 4 Mn)U( n IN

⎪⎩⎨

∈+=+ INn,UU n1n

0 ⎪⎧ =

2

2U

GnuKmn_én . ..U

k¿cUrKNna nU Ca n

x¿KNnaplKuN n21n U..UUP 0 ××××= . 49-eK[s MnYnBit kMnt´elI eday ½IVúténc )U( n IN

22U0 = ig n INn,

2U11

U 1n =+

2n ∈∀

−−

0-eK[sIVúténcMnYnBit kMnt´eday½

=

2a,INn,

au0

KNna nU CaGnuKmn_én . n

5 )u( n

⎩ −=+ 2uu 2n1n

⎨⎧

>∈∀

Page 19: HU NSEN 3 STR O N G · Five Hundred Mathematical Challenges,Complex Numbers From A to … Z , Mathematical Olympiad in China,Mathematical Olympiad Challenges, Mathematical Olympiad

KNitviTüaCMuvijBiPBelak

- 12 - PaK1

cUrRsaybBa¢ak´fa nn

4aau ⎟⎞

⎜⎛ −

+⎜⎛ +

=2

22

2

n 2a

24a

⎟⎠

⎜⎝

−⎟⎟⎠

⎞⎜⎝

51-k¿cUrkMntelxénGBaØat éncMnYn d,c,b,a abcd ebIeKdwgfa ½

dcbabcd = a9×

cMnYn x¿cMeBa¼témø EdlánrkeXIjxagelIcUrbBa¢ak´fa d,c,b,a

abcd nig dcba suTæEtCakaer®ákd . 52-snμtfa HUNSEN Cac YnmYymanelxRáMmYyxÞg´Edl Mn

Caelx . KuNcMnYne eKTTYl CacMnYnmanelx

Þg´KW

N,E,S,N,U,H

eBlEdleK n¼nwg Tæpl3

RáMmYyx STRONG Edl G,N,O,R,T,S Caelx cUrrkcMnYn HUNSEN nig STRONG cMeBa¼ 3S,6N == nigcMeBa¼ 8N 9S, == . (GkßrxusKñatagedayelxxusKña ). 53-cUrbMeBjRbmaNviFIbUkxageRkam ½

Page 20: HU NSEN 3 STR O N G · Five Hundred Mathematical Challenges,Complex Numbers From A to … Z , Mathematical Olympiad in China,Mathematical Olympiad Challenges, Mathematical Olympiad

KNitviTüaCMuvijBiPBelak

- 13 - PaK1

YENOM EROM

E

(GkßrxusKñatagedayelxxusKYn ´Ed

DNS+

ña ) 54-eK[ACacMn mYymanelxRáMxÞg lelxxÞg´vaerobtam lMdab´ x1x;x ;1x;2x; ++ CacMnYnmYymanelx

lelxxÞg+ ehIy B

RáMmYyxÞg´Ed ´vaerobtamlMdab´ x;1x;1x;2x;1x;x −+ .

cUrbgHajfae CakaerRákdena¼ B k¾Cakaer®ákdEdr .++

bI A 55-eK[cMnYn

)elxn2(

111.....111A = nig B) elxn(

444.....444=

++ CakaerRákd . MnYnmYymanelxbYnxÞg´EdlelxxÞg´vaerobtamlMdab´

.

7-eKman

cUrbgHajfa A 1B

56-c b;b;a;a

rkcMnYnena¼ebIeKdwgfavaCakaerRákd . 5 111088893333,110889333,108933 222 ===

Page 21: HU NSEN 3 STR O N G · Five Hundred Mathematical Challenges,Complex Numbers From A to … Z , Mathematical Olympiad in China,Mathematical Olympiad Challenges, Mathematical Olympiad

KNitviTüaCMuvijBiPBelak

- 14 - PaK1

333332 1111088889= . ahrN_xagelIcUrrkrUbmnþTUeTAnigRsaybBa¢ak´rUbmnþen¼pg. BI«T

58-eKman 999800019999,998001999,980199 222 === . 9999800001999992 =

BI«TahrN_xagelIcUrrkrUbmnþTUeTAnigRsaybBa¢ak´rUbmnþen¼pg. 59-eKman 444355566666,443556666,435666 222 ===

4556,115 22222 =

4444355556666662 = . BI«TahrN_xagelIcUrrkrUbmnþTUeTAnigRsaybBa¢ak´rUbmnþen¼pg 60- eKman

111111445556,111162 −=−=

«TahrN_xagelIcUrrkrUbmnþTUeTA nigRsaybBa¢ak´rUbmnþen¼pg

1111111144455556 22 =− . BI

Page 22: HU NSEN 3 STR O N G · Five Hundred Mathematical Challenges,Complex Numbers From A to … Z , Mathematical Olympiad in China,Mathematical Olympiad Challenges, Mathematical Olympiad

KNitviTüaCMuvijBiPBelak

- 15 - PaK1

61-eK[ 011nn aa..........aaA = − nig 0a2B ×− 11nn a.........aa= −

ac´nwg . Rsayfa A Eckdac´nwg 7 lu¼RtaEt B Eckd 7

62-eK[cMnYn 012nn aaa......aaA − 1=

,

nYn ac´nwg kalNa a)a...

Edl n210 a....,aa,a Caelx . .,

cUrRsayfacM A Eckd 6

0n21 aa(4y ++ Eckdacnwg nig Im,I[cMnYn

++= . 6

63-cUrkMnt´elx b eda abba CaKUbéncMnYnKt 64-eKtag r nig R erogKñaCakaMrgVg´carikkñúg nigcarikeRkA énRtIekaN ABCmYy . k¿cUrRsayfa

Rr1CcosBcosAcos +=++

. x¿cUrRsayfa R ≥ r2

65-k¿cUrRsayfa x2cotxcotx2sin

1−=

¿cUrKNna xn2

n

2a

sin

1....

2a

sin

1

2a

sin

1asin

1S ++++=

Page 23: HU NSEN 3 STR O N G · Five Hundred Mathematical Challenges,Complex Numbers From A to … Z , Mathematical Olympiad in China,Mathematical Olympiad Challenges, Mathematical Olympiad

KNitviTüaCMuvijBiPBelak

- 16 - PaK1

66-k¿cUrRsayfa xcot2x

cot11+

xcos=

¿KNn uN x aplK )

2a

cos

11).a

cos

11)(a

cos

11)(

ac1(Pn +++= ....(

22os1

n2

+

Bit n

67-eK[sIVúténcMnYn )U( kMnt´elI IN eday ½

2U0 = nig 2

INn,2

U11Un+

2n

1 ∈∀−−

= n_én .

cUrKNna KNna nU CaGnuKm n

68-9π7

cos9π4

cos9π

cosS 333 +−= 69-eK[RtIekaN mYymanmMukñúgCamMuRsYc . ABC

k¿cUrRsayfa Ctan.Btan.AtanBtanAtan Ctan =++ x¿TajbBa¢akfa 33CtanBtanAtan ≥++ .

½

70-KNnaplbUk

)9n(999......999....999999Sn

elx++++=

IeKdwgfacMnYn ½ 71-cUrkMnt´RKb´KUtémøKt´viC¢man ) ebb,a(

1bab2

a32 +

CacMnYnKt´viC¢manE2

−dr .

Page 24: HU NSEN 3 STR O N G · Five Hundred Mathematical Challenges,Complex Numbers From A to … Z , Mathematical Olympiad in China,Mathematical Olympiad Challenges, Mathematical Olympiad

KNitviTüaCMuvijBiPBelak

- 17 - PaK1

72-cUrbgHajfa ½ 1

ab8cca8bbc8a 222≥

cba+

++

++

cMeBa¼RK b´cMnYnBitviC¢man .

YnBitEdlc,b,a

73-eK[ )a( n CasIVúténcMn 21a1 =

nigcMeBa¼RKb´cMnYnKt´v ¢mann eyIgmiC an 1aa

a2

2n

+−= a

nn1n+

´cMnYnKt n eyIgman ½n

cUrRsaybBa¢ak´facMeBa¼RKb ´viC¢man aaa 321 1a......... <++++ .

øKt´ ebIeKdwgfacMeBa¼RKb´ MnYnKt´viC¢man eKman

74-cUrkMnt´RKb´KUtém 3n,m ≥

c a1aa1aa

2n

m

−+−+ CacMnYnKt´ .

YnKt´viC¢man Edl =+75-eK[bIcMn ac,b,a 10cb+ . n cUrrktémøFMbMputé cbaP ××= .

76-eKmansmPaB ba

1b

xcosa

xsin 44

+=+

Edl ,0a 0ba,0b ≠+≠ ≠

cUrbgHajfa 33

8

3

8

)ba(1+b

xcosa

xsin=+ .

Page 25: HU NSEN 3 STR O N G · Five Hundred Mathematical Challenges,Complex Numbers From A to … Z , Mathematical Olympiad in China,Mathematical Olympiad Challenges, Mathematical Olympiad

KNitviTüaCMuvijBiPBelak

- 18 - PaK1

77-eK[RtIekaN ABC mYYy . bgHajfaebI

3Ctan,

3Btan tan,

3A

ax2Ca¦ssmIkar cbxx:)E( 3 0=+++ án ena¼eK cb3a =+3 + .

atémø®ákdén 78-k¿ cUrKNn10

sin π nig 10

cos π a x¿ cUrRsayf

10sin)yx(4)yx(x 22222 π

+≤−+ RKb´cMnYn IRy,x ∈ .

79- eK[sIVúténcMnYnBit kMnt´)U( n eday4

nsin.2Un

n π=

ðajfa Edl *In∈ . N

k-cUrbg4

nsin4

ncos4

)1n(cos.2 π−

π=

π+ x-Taj[ánfa

4)1n(c1 os)2(ncos)2(U nn

n 4π+

−π

= bUk ½

U.........U

+

K-KNnapln321n UUS ++++= _éCaGnuKmn n .

n32 ≥ . n

80-eK[ n cMnYnBit a1 0a.......;;a;a;

eKtag *INn,n

a.....aaaS 1

nn32 ∈∀

++++ =

Page 26: HU NSEN 3 STR O N G · Five Hundred Mathematical Challenges,Complex Numbers From A to … Z , Mathematical Olympiad in China,Mathematical Olympiad Challenges, Mathematical Olympiad

KNitviTüaCMuvijBiPBelak

- 19 - PaK1

cUrRsayfaebI 1n1nn3211n aa.......a.a.aS +

++ ≥ ena¼eKán n

n31 a.....a≥ . 2n aaS

gHajfa 81-cUrb101

10099......

65.

43.

21

< 151<

82-eK[ cMnYnB 1n2 b.;b;b;a.....;;a .

2n

22

21

2n

222nn2211 ++++++≤+++

83-eK[ nig CasIVútcMnYnBitkMnt´elI eday nigTMnak´TMngkMenIn ½

nig RKb´ cUrKNna nig CaGnuKmn_én .

n2 it 21 ...;;a n

cUrRsayfa ½ )b.....bb)(a....aa()ba....baba( 21

)x( n )y( n IN

1y,5x 00 ==2

nn3

n1n yx3xx +=+3

nn2

n1n yyx3y +=+ INn∈

nx ny n

Page 27: HU NSEN 3 STR O N G · Five Hundred Mathematical Challenges,Complex Numbers From A to … Z , Mathematical Olympiad in China,Mathematical Olympiad Challenges, Mathematical Olympiad

KNitviTüaCMuvijBiPBelak

EpñkdMeNa¼Rsay

- 20 - PaK1

Page 28: HU NSEN 3 STR O N G · Five Hundred Mathematical Challenges,Complex Numbers From A to … Z , Mathematical Olympiad in China,Mathematical Olympiad Challenges, Mathematical Olympiad

KNitviTüaCMuvijBiPBelak

lMhat´TI1 BIcMnYnBitviC¢man nig epÞógpÞat´TMnak´TMngx y 11y3x4 =+ cUrkMntrtémøGtibrmaénGnuKmn_ ½

)y2x3)(7y)(6x()y,x(f +++= dMeNa¼Rsay kMnt´rtémøGtibrmaénGnuKmn_ ½

)y2x3)(7y)(6x()y,x(f +++= tamvismPaB eyIgán ½ GMAM −

3

3

)y,x(f3

13y3x4

)y2x3)(7y)(6x(3

)y2x3()7y()6x(

≥++

+++≥+++++

eKTaj 3

313y3x4)y,x(f ⎟

⎠⎞

⎜⎝⎛ ++

≤ eday 11y3x4 =+

eKán 5123

1311)y,x(f3

=⎟⎠⎞

⎜⎝⎛ +

≤ . dUcen¼témøGtibrmaénGnuKmn_ )y,x(f es μInwg . 512

- 21 - PaK1

Page 29: HU NSEN 3 STR O N G · Five Hundred Mathematical Challenges,Complex Numbers From A to … Z , Mathematical Olympiad in China,Mathematical Olympiad Challenges, Mathematical Olympiad

KNitviTüaCMuvijBiPBelak

- 22 - PaK1

lMhat´TI2 cUrkMntRKb´témø kñúgcenøa¼ x [

2;0] π edaydwgfa ½

24xcos13

xsin13

=+

+−

dMeNa¼Rsay kMnt´RKb´témø kñúgcenøa¼ x [

2;0] π

)1(24xcos13

xsin13

=+

+−

eKBinitü ⎟⎠⎞

⎜⎝⎛ π

−π

43sin

12sin

4

2621.

22

22.

23

3cos

4sin

4cos

3sin

−=−=

ππ−

ππ=

ehIy ⎟⎠⎞

⎜⎝⎛ π

−π

43cos

12cos

4

2622.

23

22.

21

2sin

3sin

4cos

3cos

+=+=

ππ+

ππ=

Page 30: HU NSEN 3 STR O N G · Five Hundred Mathematical Challenges,Complex Numbers From A to … Z , Mathematical Olympiad in China,Mathematical Olympiad Challenges, Mathematical Olympiad

KNitviTüaCMuvijBiPBelak

smIkar GacsresreTACa ½ )1(

x2sinx12

sin

xcosxsin212

cosxsinxcos12

sin

2xcos

12cos

xsin12

sin

2xcos

426

xsin4

26

2xcos22

13

xsin22

13

=⎟⎠⎞

⎜⎝⎛ +π

=

π

+

π

=

+

+

=

+

+

eday 2

x0 π<< ena¼eKTaján

⎢⎢⎢⎢

−π=+π

=+π

x2x12

x2x12

eKTaj 12

x π= ¦

3611x π

= ( epÞógpÞat´nwglkçx&NÐEdl[ ) dUcen¼ }

3611;

12{x ππ

∈ .

- 23 - PaK1

Page 31: HU NSEN 3 STR O N G · Five Hundred Mathematical Challenges,Complex Numbers From A to … Z , Mathematical Olympiad in China,Mathematical Olympiad Challenges, Mathematical Olympiad

KNitviTüaCMuvijBiPBelak

- 24 - PaK1

lMhat´TI3 enAkñúgRtIekaN mYyEdl ABC cAB,bAC,aBC === cUrbgHajfa

cba

2Asin

+≤ rYcsresrTMnak´TMngBIreTotEdl

RsedogKña .

dMeNa¼Rsay bgHajfa

cba

2Asin

+≤

tamRTwsþIbTsIunUs R2Csin

cBsin

bAsin

a===

Edl R CakaMrgVg´carwkeRkARtIekaN . ABC

eKTaj CsinR2c,BsinR2b,AsinR2a === eKman

CsinR2BsinR2AsinR2

cba

+=

+

2CBcos)

2A

2sin(

2Acos

2Asin

cba

2CBcos

2CBsin2

2Acos

2Asin2

CsinBsinAsin

cba

−−

π=

+

−+=

+=

+

Page 32: HU NSEN 3 STR O N G · Five Hundred Mathematical Challenges,Complex Numbers From A to … Z , Mathematical Olympiad in China,Mathematical Olympiad Challenges, Mathematical Olympiad

KNitviTüaCMuvijBiPBelak

( )1

2CBcos

2Asin

2CBcos

2Acos

2Acos

2Asin

cba

−=

−=

+

eday π<−≤ |CB|0 ena¼ 12

CBcos0 ≤−

<

tam eKTaj )1(2Asin

2CBcos

2Asin

cba

≥−

=+

.

dUcen¼ cb

a2Asin

+≤ .

TMnak´TMngBIreTotEdlRsedogKñaen¼KW ½

acb

2Bsin

+≤ nig

bac

2Csin

+≤ .

- 25 - PaK1

Page 33: HU NSEN 3 STR O N G · Five Hundred Mathematical Challenges,Complex Numbers From A to … Z , Mathematical Olympiad in China,Mathematical Olympiad Challenges, Mathematical Olympiad

KNitviTüaCMuvijBiPBelak

- 26 - PaK1

lMhat´TI4 eKyk tag[cenøa¼ I ]

4;

4[ ππ−

. cUrkMnt´GnuKmn_ f kMnt´elI [ ebIeKdwgfa ]1,1 xcosxsin)x2(sinf += rYcsRmYl f cMeBa¼RKb´ x kñúgcenøa¼ I . )x(tan2

dMeNa¼Rsay kMnt´GnuKmn_ f eKman xcosxsin)x2(sinf += elICakaer f 22 )xcosx(sin)x2(sin +=

x2sin1)x2(sinfxcosxcosxsin2xsin)x2(sinf

2

222

+=

++=

eday )x4

sin(2xcosxsin +π

=+ cMeBa¼RKb´ Ix∈

eKán 0)x2(sinf > . ehtuen¼ x2sin1)x2(sinf += tag x2sinz = dUcen¼ z1)z(f += cMeBa¼ 1z1 <<− . müa¨geTotRKb´ ]

4,

4[x ππ−∈ eKman 1xtan1 ≤≤−

Page 34: HU NSEN 3 STR O N G · Five Hundred Mathematical Challenges,Complex Numbers From A to … Z , Mathematical Olympiad in China,Mathematical Olympiad Challenges, Mathematical Olympiad

KNitviTüaCMuvijBiPBelak

naM[ . 1xtan0 2 ≤≤

dUcen¼ xcos

1xcos

1xtan1)x(tanf 222 ==+= .

lMhat´TI5 cUrbgHajfa atana2tana3tanatana2tana3tan =−− Edl

2ka π

≠ RKb´cMnYnKtrWTLaTIhV k .

dMeNa¼Rsay bgHajfa atana2tana3tanatana2tana3tan =−− eKman )aa2tan(a3tan +=

atana2tanatana2tana3tana3tan

atana2tan)atana2tan1(a3tanatana2tan1atana2tana3tan

+=−+=−

−+

=

dYcen¼ atana2tana3tanatana2tana3tan =−− .

- 27 - PaK1

Page 35: HU NSEN 3 STR O N G · Five Hundred Mathematical Challenges,Complex Numbers From A to … Z , Mathematical Olympiad in China,Mathematical Olympiad Challenges, Mathematical Olympiad

KNitviTüaCMuvijBiPBelak

- 28 - PaK1

lMhat´TI6 cUrbgHajfa ( )2

ab21xcos

b1xsin

a1 +≥⎟⎠⎞

⎜⎝⎛ +⎟

⎠⎞

⎜⎝⎛ +

cMeBa¼RKb´ 2

x0,0b,0a π<<>>

dMeNa¼Rsay bgHajfa ( )2

ab21xcos

b1xsin

a1 +≥⎟⎠⎞

⎜⎝⎛ +⎟

⎠⎞

⎜⎝⎛ +

eKman xcosxsin

abxcos

bxsin

a1)xcos

b1()xsin

a1( +++=++ tamvismPaB eKman ½ GMAM −

xcosxsinab2

xcosb

xsina

≥+

eKán xcosxsin

abxcosxsin

ab21)xcos

b1)(xsin

a1( ++≥++

22

2

)ab21()x2sin

ab21()xcos

b1()xsin

a1(

)xcosxsin

ab1()xcos

b1)(xsin

a1(

+≥+≥++

+≥++

BIeRBa¼RKb´ 2

x0 π<< eKman 1x2sin ≤ .

Page 36: HU NSEN 3 STR O N G · Five Hundred Mathematical Challenges,Complex Numbers From A to … Z , Mathematical Olympiad in China,Mathematical Olympiad Challenges, Mathematical Olympiad

KNitviTüaCMuvijBiPBelak

- 29 - PaK1

lMhat´TI7 eK[ctuekaNEkg mYyEdl BCABCD AB3= . P nig Q CaBIrcMnucénRCug Edl]BC[ QCPQBP == . cUrbgHajfa DPCDBCDQC ∠+∠=∠ .

dMeNa¼Rsay bgHajfa DPCDBCDQC ∠+∠=∠ tag γ=∠β=∠α=∠ DQC;DPC;DBC

eyIgman 1tan,21tan,

31tan =γ=β=α

eday γ==−

+=

βα−β+α

=β+α tan1

61131

21

tantan1tantan)tan(

dUcen¼ α ¦ γ=β+ DPCDBCDQC ∠+∠=∠ .

A

B C

D

P Q

Page 37: HU NSEN 3 STR O N G · Five Hundred Mathematical Challenges,Complex Numbers From A to … Z , Mathematical Olympiad in China,Mathematical Olympiad Challenges, Mathematical Olympiad

KNitviTüaCMuvijBiPBelak

- 30 - PaK1

lMhat´TI8 eK[ nig CacMnYnkMupøicBIr . 1z 2z

cUrRsayfa ( )22

21

221

221 |z||z|2|zz||zz| +=−++

dMeNa¼Rsay Rsayfa ( )2

22

12

212

21 |z||z|2|zz||zz| +=−++ eKman )zz)(zz(|zz| 2121

221 ++=+

( )1|z|zzzz|z||zz|

| zzzzzzzz|zz2

221212

12

21

222121112

21

+++=+

+++=+

ehIy )zz)(zz(|zz| 21212

21 −−=−

( )2|z|zzzz|z||zz|zzzzzzzz|zz|

222121

21

221

222121112

21

+−−=−

+−−=−

bUkTMnak´TMng ( nig )1 ( )2 eKán ½ ( )2

22

12

212

21 |z||z|2|zz||zz| +=−++ .

Page 38: HU NSEN 3 STR O N G · Five Hundred Mathematical Challenges,Complex Numbers From A to … Z , Mathematical Olympiad in China,Mathematical Olympiad Challenges, Mathematical Olympiad

KNitviTüaCMuvijBiPBelak

- 31 - PaK1

lMhat´TI9 eK[ nig CacMnYnkMupøicBIrEdl 1z 2z 1|z||z| 21 == nig z . 1z. 21 −≠

cUrRsayfa 21

21

zz1zz

++ CacMnYnBitmYy .

dMeNa¼Rsay Rsayfa

21

21

zz1zz

++ CacMnYnBitmYy

tag 21

21

zz1zzZ ena¼

++

=21

21

z.z1zzZ

++

=

eday 1|z|z.z 2111 == ena¼

11 z

1z = ehIydUcKña 2

2 z1z =

eKán Z1zz

zz

z1.

z11

z1

z1

Z12

12

21

21 =+

+=

+

+=

eday ZZ = ena¼ 21

21

zz1zzZ

++

= CacMnYnBit .

Page 39: HU NSEN 3 STR O N G · Five Hundred Mathematical Challenges,Complex Numbers From A to … Z , Mathematical Olympiad in China,Mathematical Olympiad Challenges, Mathematical Olympiad

KNitviTüaCMuvijBiPBelak

- 32 - PaK1

lMhat´TI10 KNnalImItxageRkam ½

4x4x1212...121212

lim4x −

−++++++→

man n ¦skaer .

dMeNa¼Rsay KNnalImIt

tag 4x

4x1212...121212limL

4xn −

−++++++=

( man ¦skaer ) n

eKán

4x4x1212...1212limL

4x1n −−+++++

=→−

( man ¦skaer ) 1n −

snμtfa 4x1212...121212)x(fn −++++++= nig 4x1212...121212)x(f n +++++++=

Page 40: HU NSEN 3 STR O N G · Five Hundred Mathematical Challenges,Complex Numbers From A to … Z , Mathematical Olympiad in China,Mathematical Olympiad Challenges, Mathematical Olympiad

KNitviTüaCMuvijBiPBelak

eKman ½

)x(f4x1212.....1212)x(f).x(f

16x1212...121212)x(f).x(f

1nnn

nn

−=−+++++=

−++++++=

eKTaj )x(f)x(f)x(f

n

1nn

−=

eKán )x(f

1lim.4x

)x(flim)x(f)4x(

)x(flimLn

4x

1n

4xn

1n

4xn →

→ −=

−=

)4(f

1.LLn

1nn −= eday 8)4(fn =

eKán 1nn L81L −= naM[ CasIVútFrNImaRtmanersug )L( n

81q = nigtY

4x4x12limL

4x1 −−+

=→

. tamrUbmnþtYTI n eKán 1n

1n qLL −×= eday

81

4x121lim

4x4x12limL

4x4x1 =++

=−

−+=

→→

nig 81q = ena¼ nn 8

1L = . dUcen¼

n4x 81

4x4x1212...121212

lim =−

−++++++→

.

- 33 - PaK1

Page 41: HU NSEN 3 STR O N G · Five Hundred Mathematical Challenges,Complex Numbers From A to … Z , Mathematical Olympiad in China,Mathematical Olympiad Challenges, Mathematical Olympiad

KNitviTüaCMuvijBiPBelak

- 34 - PaK1

lMhat´TI11 eK[sIVúténcMnYnBit kMnt´eday ½ )u( n

1u0 = nig 1u4u6u4

uu RKb´ n

2n

3n

4n

1n +++=+ INn∈

cUrbgHajfa 4

n1n u11

u11 RKb´ ⎟⎟

⎞⎜⎜⎝

⎛+=+

+

INn∈

rYcKNna u CaGnuKmn_én . n n

dMeNa¼Rsay

bgHajfa 4

n1n u11

u11 ⎟⎟

⎞⎜⎜⎝

⎛+=+

+

eyIgman 4n

n2

n3

n

1n u1u4u6u41

u11 +++

+=++

4

n4

n

4n

4n

n2

n3

n4

n

u11

u)1u(

u1u4u6u4u

⎟⎟⎠

⎞⎜⎜⎝

⎛+=

+=

++++=

dUcen¼ 4

n1n u11

u11 ⎟⎟

⎞⎜⎜⎝

⎛+=+

+

.

Page 42: HU NSEN 3 STR O N G · Five Hundred Mathematical Challenges,Complex Numbers From A to … Z , Mathematical Olympiad in China,Mathematical Olympiad Challenges, Mathematical Olympiad

KNitviTüaCMuvijBiPBelak

KNna CaGnuKmn_én nu n

eyIgman 4

n1n u11

u11 ⎟⎟

⎞⎜⎜⎝

⎛+=+

+

eKán ⎟⎟⎠

⎞⎜⎜⎝

⎛+=⎟⎟

⎞⎜⎜⎝

⎛+

+ n1n u11ln4

u11ln

tag ⎟⎟⎠

⎞⎜⎜⎝

⎛+=

nn u

11lnv naM[ ⎟⎟⎠

⎞⎜⎜⎝

⎛+=

++

1n1n u

11lnv

eKán . n1n v4v =+

TMnak´TMngen¼bBa¢ak´fa CasIVútFrNImaRtmanersug )v( n 4q = nigtY 2ln

u11lnv

00 =⎟⎟

⎞⎜⎜⎝

⎛+= ( eRBa¼ 1u0 = ) .

tamrUbmnþ 2ln4q.vv nn0n == .

eday ⎟⎟⎠

⎞⎜⎜⎝

⎛+=

nn u

11lnv

eKTaj 2ln4u11ln n

n

=⎟⎟⎠

⎞⎜⎜⎝

⎛+ naM[ n4

n

2u11 =+

dUcen¼ 12

1u n4n−

= .

- 35 - PaK1

Page 43: HU NSEN 3 STR O N G · Five Hundred Mathematical Challenges,Complex Numbers From A to … Z , Mathematical Olympiad in China,Mathematical Olympiad Challenges, Mathematical Olympiad

KNitviTüaCMuvijBiPBelak

lMhat´TI12 KNna

)xx1).....(xx1)(xx1(P1nn 22422

n

++−+−+−=

dMeNa¼Rsay KNna

)xx1).....(xx1)(xx1(P1nn 22422

n

++−+−+−=

eKman )aa1)(aa1(a)a1(aa1 2222242 +++−=−+=++ eKTaj 2

422

aa1aa1aa1

++++

=+− yk k2xa =

eKán 1kk

2k1k1kk

22

2222

xx1xx1xx1 +

+++

++

++=+−

2

22n

0k 22

22

n xx1xx1

xx1xx1P

2n1n

1kk

2k1k

++++

=⎟⎟⎠

⎞⎜⎜⎝

++

++=

++

+

++

∏=

dUcen¼ 2

22

n xx1xx1P

2n1n

++++

=++

.

- 36 - PaK1

Page 44: HU NSEN 3 STR O N G · Five Hundred Mathematical Challenges,Complex Numbers From A to … Z , Mathematical Olympiad in China,Mathematical Olympiad Challenges, Mathematical Olympiad

KNitviTüaCMuvijBiPBelak

- 37 - PaK1

lMhat´TI13 eK[RtIekaN mYymanRCugRbEvg . ABC c,b,a

knø¼bnÞat´Bu¼énmMu C kat´ [ Rtg´cMnuc D . ]AB

cUrRsayfa 2Ccos

baab2CD+

= .

dMeNa¼Rsay Rsayfa

2Ccos

baab2CD +

=

eyIgman S CDBACDABC SS += eday Csinab

21SABC =

2CsinCDb

21SACD = nig

2CsinCDa

21SCDB =

eKán 2CsinCD.a

21

2CsinCD.b

21sibCab

21

+= eKTaj

2Ccos

baab2

2Csin)ba(

CsinabCD+

=+

= .

dUcen¼ 2Ccos

baab2CD+

= .

C

A D B

Page 45: HU NSEN 3 STR O N G · Five Hundred Mathematical Challenges,Complex Numbers From A to … Z , Mathematical Olympiad in China,Mathematical Olympiad Challenges, Mathematical Olympiad

KNitviTüaCMuvijBiPBelak

- 38 - PaK1

lMhat´TI14 KNnaplbUk

!n.n.....!3.3!2.2!1.1Sn ++++= Edl 1.2.3)....2n)(1n(n!n −−= .

dMeNa¼Rsay KNnaplbUk

!n.n.....!3.3!2.2!1.1Sn ++++= eKman !k.]1)1k[(!k.k −+=

!k)!1k(!k.k!k!k).1k(!k.k

−+=−+=

eKán !n)!1n(...)!3!4()!2!3()!1!2(Sn −+++−+−+−= dUcen¼ 1)!1n(Sn −+= .

Page 46: HU NSEN 3 STR O N G · Five Hundred Mathematical Challenges,Complex Numbers From A to … Z , Mathematical Olympiad in China,Mathematical Olympiad Challenges, Mathematical Olympiad

KNitviTüaCMuvijBiPBelak

lMhat´TI15 KNnatémø

)29tan3)...(3tan3)(2tan3)(1tan3(A oooo ++++=

dMeNa¼Rsay KNnatémø A

)29tan3)...(3tan3)(2tan3)(1tan3(A oooo ++++= eKman oo

ooo0

1cos60cos61sin1tan60tan1tan3 =+=+

oo

ooo0

oo

oooo

oo

oooo

29cos60cos89sin29tan60tan29tan3

3cos60cos63sin3tan60tan3tan3

2cos60cos62sin2tan60tan2tan3

=+=+

−−−−−−−−−−−−−−−−−−−−−−−−

=+=+

=+=+

eKán 29ooooo29o

oooo

)60(cos1

29cos...3cos2cos1cos)60(cos89sin....63sin.62sin.61sinA ==

eday 2160cos o = dUcen¼ 5368709122A 29 == .

- 39 - PaK1

Page 47: HU NSEN 3 STR O N G · Five Hundred Mathematical Challenges,Complex Numbers From A to … Z , Mathematical Olympiad in China,Mathematical Olympiad Challenges, Mathematical Olympiad

KNitviTüaCMuvijBiPBelak

- 40 - PaK1

lMhat´TI16 eK[ CacMnYnBitEdlepÞógpÞat´lkçx&NÐ ½ z,y,x

0zcosycosxcos =++ nig cos 0z3cosy3cosx3 =++ cUrbgHajfa cos 0z2cosy2cosx2 ≤ . dMeNa¼Rsay bgHajfa cos 0z2cosy2cosx2 ≤ tamrUbmnþ cos acos3acos4a3 3 −= eKTaj )1(x3cosxcos3xcos4 3 +=

)3(z3coszcos3zcos4)2(y3cosycos3ycos4

3

3

+=

+=

bUkTMnak´TMng eKán ½ )3(,)2(,)1(

0zcos4ycos4xcos4 333 =++

¦ 0zcosycosxcos 333 =++ (eRBa¼ 0zcosycosxcos =++ nig 0z3cosy3cosx3cos =++ ) eKman 0zcosycosxcos =++ eKán zcosycosxcos −=+

Page 48: HU NSEN 3 STR O N G · Five Hundred Mathematical Challenges,Complex Numbers From A to … Z , Mathematical Olympiad in China,Mathematical Olympiad Challenges, Mathematical Olympiad

KNitviTüaCMuvijBiPBelak

elIkCaKUbeKán ½

zcosycosxcos3xcosycosxcoszcosycoszcosycosxcos3xcos

zcosycos)ycosx(cosycosxcos3xcoszcos)ycosx(cos

333

333

333

33

=++

−=+−

−=+++

−=+

eday 0zcosycosxcos 333 =++ eKTaj 0zcosycosxcos3 = naM[ ¦ 0xcos = 0ycos = ¦ 0zcos = . edaysn μtyk 0xcos = ena¼ zcosycos −= eKán ½

0)1zcos2(z2cosy2cosx2cos)1zcos2)(1ycos2)(1xcos2(z2cosy2cosx2cos

22

222

≤−−=

−−−=

dUcen¼bBaHaRtUvánRsaybBa¢ak´ .

- 41 - PaK1

Page 49: HU NSEN 3 STR O N G · Five Hundred Mathematical Challenges,Complex Numbers From A to … Z , Mathematical Olympiad in China,Mathematical Olympiad Challenges, Mathematical Olympiad

KNitviTüaCMuvijBiPBelak

- 42 - PaK1

lMhat´TI17 eK[RtIekaN mYy . bnÞatBu¼mMu kat´RCug ABC C,B,A

]AB[,]AC[,]BC[ erogKñaRtg´ . 'C,'B,'A

cUrRsayfa ½

0'CC

)2

BAsin(

'BB

)2

ACsin(

'AA

)2

CBsin(=

+

+

dMeNa¼Rsay

eyIgman S B'CC'ACCABC SS += eday Csinab

21SABC =

2Csin'CCb

21S 'ACC = nig

2Csin'CCa

21S B'CC =

eKán 2Csin'CC.a

21

2Csin'CC.b

21sibCab

21

+=

C

A 'C B

Page 50: HU NSEN 3 STR O N G · Five Hundred Mathematical Challenges,Complex Numbers From A to … Z , Mathematical Olympiad in China,Mathematical Olympiad Challenges, Mathematical Olympiad

KNitviTüaCMuvijBiPBelak

eKTaj )1(2Ccos

baab2

2Csin)ba(

Csinab'CC+

=+

=

eKman 2Ccos

2Csin2

2BAcos

2BAsin2

CsinBsinAsin

cba

+−

=−

=−

eday 2Csin

2C

2cos

2BAcos =⎟

⎠⎞

⎜⎝⎛ −π

=+

eKán 2Ccos2

BAsin

cba

=− naM[ )

2BAsin(.

bac

2Ccos −

−=

tam eKán )1( )2

BAsin(ba

abc2'CC 22

−−

=

eKTaj abc2

ba'CC

)2

BAsin( 22 −=

.

dUcKñaEdr abc2

ac'BB

)2

ACsin(;

abc2cb

'AA

)2

CBsin( 2222 −=

−−

=

dUcen¼ 0'CC

)2

BAsin(

'BB

)2

ACsin(

'AA

)2

CBsin(=

+

+

.

- 43 - PaK1

Page 51: HU NSEN 3 STR O N G · Five Hundred Mathematical Challenges,Complex Numbers From A to … Z , Mathematical Olympiad in China,Mathematical Olympiad Challenges, Mathematical Olympiad

KNitviTüaCMuvijBiPBelak

- 44 - PaK1

lMhat´TI18 eK[BhuFa P n)acosasinx()x( += Edl *INn∈ cUrrksMNlénviFIEckrvag nwg x)x(P 12 + .

dMeNa¼Rsay sMNl´énviFIEck tag )x(R CasMNl´énviFIEckrvag nwg )x(P 1x2 +

-ebI ena¼ 1n = acosasinx)x(P += dUcen¼ acosasinx)x(R += CasMnl´énviFIEck . -ebI eKán 2n ≥ )x(R)x(Q)1x()x(P 2 ++= Edl CaplEck nwg )x(Q BAx)x(R += eKán BAx)x(Q)1x()acosasinx( 2n +++=+ ebI ena¼ ix = BAi)acosasini( n +=+ ¦ A.iB)nasin(.i)nacos( +=+ eKTaj nig )nasin(A = )nacos(B = dUcen¼ )nacos()nasin(x)x(R += .

Page 52: HU NSEN 3 STR O N G · Five Hundred Mathematical Challenges,Complex Numbers From A to … Z , Mathematical Olympiad in China,Mathematical Olympiad Challenges, Mathematical Olympiad

KNitviTüaCMuvijBiPBelak

- 45 - PaK1

lMhat´TI19

eda¼RsayRbB&næsmIkar ⎩⎨⎧

−=−

+=+

byaxyxbayx

44

3

Edl nig b CacMnYnBitEdl aa 0b ≠+ .

dMeNa¼Rsay eda¼RsayRbB&næsmIkar

⎩⎨⎧

−=−

+=+

byaxyxbayx

44

3

smmUl 1y

yxbyax)yx(ba

44

3

⎩⎨⎧

−=−

+=+

443

44

3

yx)yx(y)yx(a

yxbyax)yx(ybyay

−++=+

+⎩⎨⎧

−=−

+=+

eday ena¼ 0ba ≠+ 0yx ≠+ eKTaj 23

443

xy3xyx

yx)yx(ya +=+

−++=

ehIy 323 yyx3a)yx(b +=−+=

eKánRbB&næ naM[ ⎩⎨⎧

=+

=+

byyx3axy3x

32

23

⎩⎨⎧

−=−

+=+3

3

bayxbayx

dUcen¼ 2

babay;2

babax . 3333 −−+

=−++

=

Page 53: HU NSEN 3 STR O N G · Five Hundred Mathematical Challenges,Complex Numbers From A to … Z , Mathematical Olympiad in China,Mathematical Olympiad Challenges, Mathematical Olympiad

KNitviTüaCMuvijBiPBelak

- 46 - PaK1

lMhat´TI20 cUrbgHajfa 9

)1nn)(1nn(19999

1n44 =⎟

⎞⎜⎝

⎛++++

∑=

dMeNa¼Rsay bgHajfa 9

)1nn)(1nn(19999

1n44 =⎟

⎞⎜⎝

⎛++++

∑=

eKman n1n)n1n)(n1n( 4444 −+=−+++

n1n1)n1n)(n1n( 4444

++=−+++

eKTaj 4444 n1n

)1nn)(1nn(1

−+=++++

9110000

)n1n()1nn)(1nn(

1

4

9999

1n

449999

1n44

=−=

−+=⎟⎠

⎞⎜⎝

⎛++++

∑∑==

dUcen¼ 9)1nn)(1nn(

19999

1n44 =⎟

⎞⎜⎝

⎛++++

∑=

.

Page 54: HU NSEN 3 STR O N G · Five Hundred Mathematical Challenges,Complex Numbers From A to … Z , Mathematical Olympiad in China,Mathematical Olympiad Challenges, Mathematical Olympiad

KNitviTüaCMuvijBiPBelak

- 47 - PaK1

lMhat´TI21 eK[GnuKmn_ kMnt´RKb´ f }0;1{IRx −−∈ eday ½

1x)x1(f)x(f)1x2(x +=++ .

cUrKNna )2009(f....)3(f)2(f)1(fS ++++=

dMeNa¼Rsay KNna )2009(f....)3(f)2(f)1(fS ++++= eKman )1(1x)

x1(f)x(f)1x2(x +=++

CMnYs eday xx1 kñúgsmIkar ( eKán ½ )1

)2(2xxx)x(f

2xx)

x1(f

1x1)x(f)

x1(f)1

x2(

x1

22

++

=+

+

+=++

dksmIkar nig Gg:nwgGg:eKán ½ )1( )2(

2x2x2)x(f

2x)1x(x2

2xxx1x)x(f

2xx)1x2(x

2

22

++

=++

++

−+=⎥⎦

⎤⎢⎣

⎡+

−+

eKTaján 1x

1x1

)1x(x1)x(f

+−=

+=

Page 55: HU NSEN 3 STR O N G · Five Hundred Mathematical Challenges,Complex Numbers From A to … Z , Mathematical Olympiad in China,Mathematical Olympiad Challenges, Mathematical Olympiad

KNitviTüaCMuvijBiPBelak

- 48 - PaK1

eKán [ ] ∑∑==

=−=⎟⎠⎞

⎜⎝⎛

+−==

2009

1k

2009

1k 20092008

200911

1k1

k1)k(fS

dUcen¼ 20092008)2009(f....)3(f)2(f)1(fS =++++= .

lMhat´TI22 cUrkMntRKb´KU éncMnYnKt´viC¢manebIeKdwgfa ½ )n;m(

)nm(13nm 22 +=+ .

dMeNa¼Rsay kMnt´RKb´KU ½ )n;m(

eKman )1()nm(13nm 22 +=+ -krNITI1 nm =

eKán naM[ n26n2 2 = 13n = dUcen¼ . 13nm ==

-krNITII2 nm ≠

eyIgBinitüeXIjfaebI ( CaKUcemøIyrbs ena¼eKán )n;m )1(

)m;n( k¾CaKUcemøIyrbs Edr . )1(

snμtfa m . tamvismPaB Cauchy Schwarz n<

Page 56: HU NSEN 3 STR O N G · Five Hundred Mathematical Challenges,Complex Numbers From A to … Z , Mathematical Olympiad in China,Mathematical Olympiad Challenges, Mathematical Olympiad

KNitviTüaCMuvijBiPBelak

eyIgman )nm(2)nm( 222 +<+

26nm)nm(26)nm( 2

<++<+

eday ena¼eKán nm < 26nmm2 <+< ¦ 13m <

eday ena¼eKTaj *INm∈ 12m1 ≤≤ . müa¨geTotsmIkar Gacsresr ½ )1(

)2(0m13mn13n 22 =−+− DIsRKImINg´ )m13m(4169 2 −−=Δ smIkar mancemøIykñúg kalNa )2( *IN Δ CakaerRákd éncMnYnKtviC©maness . eKyk INk)1k2()m13m(4169 22 ∈∀+=−− eKán )1k(k41)1k2()m13m(4168 22 +=−+=−− eKTaj )m13(m42)1k(k −+=+ eday 12m1 ≤≤ ena¼témøEdlGacrbs´plKuN )1k(k + KW ½

}84,82,78,72,64,54{)1k(k =+ . kñúgtémøTaMgRáMmYyen¼témøEdlCaplKuNcMnYnKt´tKñamanEt témø mYyKt´EdlRtUvnwg 9872 ×= }10;3{m = .

- 49 - PaK1

Page 57: HU NSEN 3 STR O N G · Five Hundred Mathematical Challenges,Complex Numbers From A to … Z , Mathematical Olympiad in China,Mathematical Olympiad Challenges, Mathematical Olympiad

KNitviTüaCMuvijBiPBelak

-cMeBa¼ eKán 3m = 0)2n)(15n(30n13n2 =+−=−− naM[ . 15n =

-cMeBa¼ eKán 10m = 0)2n)(15n(30n13n2 =+−=−− naM[ . 15n =

srubmkeKTTYlánKUcemøIy®áMKUKW ½ })10;15(;)3;15(;)13;13(;)15;10();15;3({)n;m( = .

- 50 - PaK1

Page 58: HU NSEN 3 STR O N G · Five Hundred Mathematical Challenges,Complex Numbers From A to … Z , Mathematical Olympiad in China,Mathematical Olympiad Challenges, Mathematical Olympiad

KNitviTüaCMuvijBiPBelak

- 51 - PaK1

lMhat´TI23 eK[RtIekaN mYyEkgRtg . eKyk ABC A ]AC[E∈

nig F Edl ]AB[∈ ABCAEF ∠=∠ nig ACBAFE ∠=∠ . eKtag nig F erogKñaCaeCIgcMeNalEkgén E nig F 'E '

elIRCug [ . ]BC

cUrRsayfa E BC'FFEFE' ≤++ ?

dMeNa¼Rsay Rsayfa E BC'FFEFE' ≤++ tag BC cAB,bAC,a === . RtIekaNEkg nig manmMu ABC AFE ABCAEF ∠=∠ vaCaRtIekaNdUcKña .

A

B C

F

E

'F 'E

Page 59: HU NSEN 3 STR O N G · Five Hundred Mathematical Challenges,Complex Numbers From A to … Z , Mathematical Olympiad in China,Mathematical Olympiad Challenges, Mathematical Olympiad

KNitviTüaCMuvijBiPBelak

eKánpleFobdMnUc kBCEF

ACAF

ABAE

=== Edl 0k > tag[pleFobdMnUc . eKTaj akEF,bkAF,ckAE === . RtIekaNEkg nig manmMu ABC C'EE ACB∠ rYmvaCaRtIekaN dUcKña . eKán

ECBC

'EEAB

= naM[ BC

EC.AB'EE = eday ckbAEACEC −=−= eKán

a)ckb(c'EE −

= . dUcKñaEdreKTaj

a)bkc(b'FF −

= eKán

a)bkc(bak

a)ckb(c'FFEFE'E −

++−

=++

abc2

a)cba(kbc2

akbbckakcbc

222

222

=−−+

=

−++−=

BIeRBa¼kñúgRtIekaNEkg eKman ABC 222 cba += (BItaK&r) tamvismPaB eKman GMAM − bc2cba 222 ≥+= eKTaj BCa

abc2

=≤ .dUcen¼ BC'FFEFE'E ≤++ .

- 52 - PaK1

Page 60: HU NSEN 3 STR O N G · Five Hundred Mathematical Challenges,Complex Numbers From A to … Z , Mathematical Olympiad in China,Mathematical Olympiad Challenges, Mathematical Olympiad

KNitviTüaCMuvijBiPBelak

- 53 - PaK1

lMhat´TI24 eK[ c,b,a CabIcMnYnBitviC¢man . cUrRsaybBa¢akfa ½

2)ba(4c

ba)ac(4b

ac)cb(4a

cb3 333 333 33

≤++

++

+++

+++

+

dMeNa¼Rsay Rsayfa ½ Rsayfa ½

2)ba(4c

ba)ac(4b

ac)cb(4a 3 +

cb3 333 3333

≤+

++

+++

++

+

)cb( 3

+

eKman b(bc3cb 33 )c+−+= PaB AM − an +

tamvism M eKmG bc2cb ≥+ eKTaj

2

2cbbc ⎟⎠⎞

⎜⎝⎛ +

≤ naM[ 3)cb(43)cb(bc3 +−≥+−

eKán 33333 )cb(41)cb(b

43)cb(c +=+−+≥+

aj eKT 3 33 )cb(4cb +≤+ ¦ 3 33 )cb(4acba + ++≤+ naM[ ( )1

cbacb

)cb(4a 3

cb33 ++

+≤

++

+

Page 61: HU NSEN 3 STR O N G · Five Hundred Mathematical Challenges,Complex Numbers From A to … Z , Mathematical Olympiad in China,Mathematical Olympiad Challenges, Mathematical Olympiad

KNitviTüaCMuvijBiPBelak

- 54 - PaK1

dUcKñaEdr ( )2cba

ac)ac(4b

ac3 33 ++

+≤

+++

nig ( )3c)ba(c

ba3 33 ba

ba4 ++

+≤

+

edaybUkTMnak´TMng eKán ½ +

+ .

)3(,)2(,)1(

2)ba(4c

ba)ac(4b

ac)cb(4a 3

cb3 333 3333

≤++

++

+++

++ .

++

lMhat´TI25 eK[sIVútcMnYn

−Bit

=⎩⎨⎧

≥∀+

==

−+ 1n,2a2aaa

1nn1n

10

2 ++

a1

cUrRsayfa a 1nn1naa ×= .

dMeNa¼Rsay a aa 2 +

Rsayf 1nn a +1n×=

a2a + −=

aa −−= RK ≥ −=

2aa2(c nn1nn1n

eKman a 1−

tag c b´ n

nn1n

1nnn

eKán c

1

n1n1n ++ aa

a) 2c +=−+−= −+

Page 62: HU NSEN 3 STR O N G · Five Hundred Mathematical Challenges,Complex Numbers From A to … Z , Mathematical Olympiad in China,Mathematical Olympiad Challenges, Mathematical Olympiad

KNitviTüaCMuvijBiPBelak

- 55 - PaK1

TMnak´TMngen¼bBa¢ak´fa CasIVútnBVnþmanplsgrYm )c( n 2d = 01nigtY 0c1 11aa =−=−= .

eKán 2n2)1n(2d)1n(cc 1n −=−=−+= 1nneKTaj a 2n2a −=− −

)

( ) (

)1n(n=1a

)12aa

1k2aa

n

0n

n

1k

n

1k1kk

−−

=−

−=− ∑∑==

2n(n −

eKTaj 1nna 2n +−= nig 1)1n()1n(a 2

1n ++−+=+ 2 ++= nig ¦ a 1n+ 1n21nn 1nna 24 ++=

+

nn(a −+eday )(1a 222221nn n)1n()1nn =+++−=+

¦ 1nnaa 241nn ++=+ .

dUcen¼

1nn1naaa 2 ++

×= .

Page 63: HU NSEN 3 STR O N G · Five Hundred Mathematical Challenges,Complex Numbers From A to … Z , Mathematical Olympiad in China,Mathematical Olympiad Challenges, Mathematical Olympiad

KNitviTüaCMuvijBiPBelak

- 56 - PaK1

lMhat´TI26 kñúgRtIekaN ABC mYycUrRsaybBa¢akfa ½

rR4111

≥ 2Csin

22Edl r nig

BsinA ++sin

R CakaMrgVg´carwkkñúg nig carwkeRkARtIekaN .

dMeNa¼

Rsay fa Rsay ( )1

r2Csin

2Bsin

2Asin

R4111++

tag ,bAC,aBCtag A,bAC,aBC

cB cBA ===

wsþIbTkUsIunUskñúgRtIekaN eKman ½ 222 − eday

tamRT ABC

Acos.bc2cba +=2Asin21Acos 2−=

ena¼ )2

sin21(bc2 − Acba 2222 −+=

eKTaj bc4

)cba)(cba()cb( 22 −−+−−bc4

a2Asin2 += =

tag 2

cbap =++ ( knø¼brimaRténRtIekaN )

eKán )cp(2cba −=−+ nig )bp(2cba −=+−

Page 64: HU NSEN 3 STR O N G · Five Hundred Mathematical Challenges,Complex Numbers From A to … Z , Mathematical Olympiad in China,Mathematical Olympiad Challenges, Mathematical Olympiad

KNitviTüaCMuvijBiPBelak

- 57 - PaK1

eKán bc

)cp)(bp(2Asin2 −−=

naM[ bc

)cp)(bp(2Asin −−= . dUcKñaEdreKTaj ½

ab)bp)(ap(

2Csin;

ac)cp)(ap(

2Bsin =

−−=

−−

eKán abc

)cp)(bp)(ap(2Cs sin

2Bsin

2Ain −−−

= eKman

R4abcpr =)cp)(bp)(ap(pS =−−−=

eKTaj S.R4ab nig c = S.rpS)cp)(bp)(ap( −

2

==−−

eKán R4r

S.R4S.r

2Csin

2Bsin

2Asin == .

vismPaB smmUleTAnwg ½ ( )1

( )22

2Csin

2Bsin

2Asin

2Bsin

2Asin

2Csin

2Asin

2Csin

2Bsin

2Csin

2Bsin

2Asin4

14

2Csin

1

2Bsin

1

2Asin

1

≥++

≥++

eday 2Asin

aap

bca)cp)(bp()si 2

−=

−−ap(2Csin

2Bn

2−=

Page 65: HU NSEN 3 STR O N G · Five Hundred Mathematical Challenges,Complex Numbers From A to … Z , Mathematical Olympiad in China,Mathematical Olympiad Challenges, Mathematical Olympiad

KNitviTüaCMuvijBiPBelak

- 58 - PaK1

eKTaj a

p −=

a

2Asin

2Csin

2Bsin

. dUcKñaEdreKTaján ½

bbp2

Asin2Csin −

2Bsin

= nig c

cp

2Csin

2Bs

2Asin in −

=

smmUleTAvismPaB ( )2 nwg ½ 2

ccp

bbp

aap

≥−

+−

+−

aB eKán ½ tamvismP GMAM −

a)ap(2a)ap(p −= −≥+ naM[ p

)ap(2a

ap −≥

dUcKñaEdr p

)bp(2b

bp −≥

− nig p

)cp(2c

cp −≥

− eKán

Bit2c

cpb

bpa

ap

)cp()bp()ap(2c

cpb

bpap

≥−

+−

+−

−+−+−≥

−+

−+

pa

dUcen¼r

4CsinBsin≥++

R

2

1

2

1

2Asin

. 1

Page 66: HU NSEN 3 STR O N G · Five Hundred Mathematical Challenges,Complex Numbers From A to … Z , Mathematical Olympiad in China,Mathematical Olympiad Challenges, Mathematical Olympiad

KNitviTüaCMuvijBiPBelak

- 59 - PaK1

lMhat´TI27 eK[ CabIcMnYnBitviC¢manEdl abc4c,b,a 1cba +++= UrbgHajfa ½ rbgHajfa ½ c

)cabcab(2c

bab

accb 222 ++

+a

222

++≥+

+

dMeNa¼Rsay Rsayfa ½

)ab(2 + cabcc

bab

aa

22

+≥+

++

aB eKman ½

ccb 2222 ++

tamvismP GMAM −

4 abcbaabc4 ++= naM[ 41c ≥+ eKTaj

1abc ≥

)1(abc31abc4cba ≥−=++ ( eRBa¼ ) 1abc ≥

tamvismPaB GMAM − eKán ½ ( )2

cab2ca2cccb 222 ++

bab2

cba

ba

a

222

++≥+

++ mPaB Cauchy - Schwarz eKman ½ tamvis

[ ]2222 )ca()bc()ab(3)cabcab( ++≤+ +

eday ])ab()ca()bc([abcca

=++2ab2

bca2 222 ++ bc2

Page 67: HU NSEN 3 STR O N G · Five Hundred Mathematical Challenges,Complex Numbers From A to … Z , Mathematical Olympiad in China,Mathematical Olympiad Challenges, Mathematical Olympiad

KNitviTüaCMuvijBiPBelak

- 60 - PaK1

eKTaj ( )3)cabcab(abc32

cab2

bca2

abc2 2++≥++

tam nig eKTaján½ )2( )3(

( )4)cabcab(abc32

cba

bac

acb 22

++ 2

2222

++≥+

++

22 +

eyIgman⎪⎩

⎪⎨

≥+

≥+

≥+

bca2)ab()ca(abc2)ca()bc(

cab2)bc()ab(

222

222

222

eKán )ab([2 2 )cba(abc2])ca()bc( +≥++ )cba(abc)ca()bc()ab( 222 ++≥++

aMgBIrnwg )()(ab(2EfmGg:T )caab(2)bc )ca)(bc(2++ a(abc3)cabc 2eKán bab( )c++≥++

≥++ ( tam eKman a abc3cb ( )1 )

naM[

eKTaj 2222 cba9)cabcab( ≥++

¦ abc3cabcab ≥++ ( )5 )cabcab(2)cabcab(

abc32 2 ++≥++

eKTaján ½ tam )4( nig )5(

)cabcab(2c

baaa

cb 22222

≥+

+++

bc2

+++ .

Page 68: HU NSEN 3 STR O N G · Five Hundred Mathematical Challenges,Complex Numbers From A to … Z , Mathematical Olympiad in China,Mathematical Olympiad Challenges, Mathematical Olympiad

KNitviTüaCMuvijBiPBelak

- 61 - PaK1

lMhat´TI28 eK[sIVút a{ ½ }n kMnteday

= nig 1a0 4a....a.aa n101n +=+ cMeBa¼RKb´ . 1n ≥

cUrbgHajfa 2aa 1nn =− + cMeBa¼RKb´ . 1n ≥

dMeNa¼Rsay bgHajfa 2aa 1nn =− +

g;teXIjfaRKb´ eyIgse INk∈ eKman . eKman

n 0ak > . eKman

0ak >

4a....a.aa n101n 4a....a.aa n101n +=+ 1n102neKán a 4a.....a.a += ++

2n102n

n0n102n

)2a.....aa(

4)a....a.4)a.....aa(

4)4a......a)(a.....a.a(

+=

++= 1

a(a

aa

2n102n

n10

a

++=

+

+

+

eKTaj 2a... n..aaa 102n +=+ 2)4a(a 1n2n +−= ++ ¦ 2aa 2n1n =− ++

dUcen¼ 2aan − 1n =+ .

Page 69: HU NSEN 3 STR O N G · Five Hundred Mathematical Challenges,Complex Numbers From A to … Z , Mathematical Olympiad in China,Mathematical Olympiad Challenges, Mathematical Olympiad

KNitviTüaCMuvijBiPBelak

- 62 - PaK1

lMhat´TI29 eK[ CabIcMnYnBitviC¢manEdl c,b,a 1cabcab =++ . cUrbgHajfa 16)

a1c()

c1b()

b1a( 222 ≥+++++

¼

dMeNa Rsay a bgHajf 16)

a1c()

c1b()

b1a( 222 ≥+++++

tag 222 )a1c()

c1b()

b1a(A = +++++

⎟⎠⎞

⎜⎝⎛ ++= a ++++++

ac

cb

ba2

c1

b1

a1cb 222

222

eyIgBinitü 22 acabcab

a1 ++= 2a

bcac

ab

++= 22

cab1

bbcab ++

= 2bac

bc

ba+= +

22 ccabcab

c1 ++= 2c

abca

cb

++= PaB eKman ½ tamvism GMAM −

3cab

bca

abc,

ab≥+ 2

ac

ca,2

bc

cb;2ba

222 ≥++≥+≥+ eKán 93222

c1

b1

a1

2 + 22 =+++≥+

Page 70: HU NSEN 3 STR O N G · Five Hundred Mathematical Challenges,Complex Numbers From A to … Z , Mathematical Olympiad in China,Mathematical Olympiad Challenges, Mathematical Olympiad

KNitviTüaCMuvijBiPBelak

- 63 - PaK1

ehIy 3a

müa¨geT c2

c.cb.

ba3

ac

cb

ba

3 =≥++ ot b,ab2ba 22222 ≥ ca2ac;bc2 +≥+≥+

222naM[ a2 ca2bc2ab2c2b2 ++≥++ aM[ n 1cabcabcba 222 =++≥++

eKán 1A 16)3(29 =+ .

+≥

dUcen¼ 16)acb

1c()1b()1a( 222 ≥+++++ .

lMhat´TI30 eK[

432 2cos2222,

2cos222,

2c22 os π

=++π

=+π IcUrrkrUbmnþTUeTA nig RsaybBa¢ak´

a¼pg .

=

BI«TahrN_xagelrUbmnþen

dMeNa¼Rsay

eKman rkrUbmnþTUeTA ½

,2

cos222,2

cos22 32

π=+

π=

42cos2222 π

=++

Page 71: HU NSEN 3 STR O N G · Five Hundred Mathematical Challenges,Complex Numbers From A to … Z , Mathematical Olympiad in China,Mathematical Olympiad Challenges, Mathematical Olympiad

KNitviTüaCMuvijBiPBelak

- 64 - PaK1

tamlMnaM«TahrN UeTAdUcxa ½_eyIgGacTajrkrUbmnþT geRkam

1n

)n(2

co222 s2.........2 +

π=+ +++ .

eyIgtagRsaybBaØak´rUbmnþen¼ ½

)n(

n 2......222A ++++= RK b´ n∈ *IN

eyIgman 21 2cos22A π

== Bit favaBitdl´tYTI W ½ eyIg«bma Kp

1p

)p(

p 2222A2

cos2...... +

π=++ Bit

ayfavaBitdl´tYTI

++=

eyIgnwgRs 1p + KW 2p1p 2cos2A ++

π= Bit

eyIgman p1p A2A +=+ aytamkar«bma ed 1pp 2

cos2A +

π=

eyIgán 2p2p

21p1pA

2cos2

2cos4

2cos22 ++++

π=

π=

π+= Bit

dUcen¼ 1ncos22.........222 +

π=++++

)n(2

.

Page 72: HU NSEN 3 STR O N G · Five Hundred Mathematical Challenges,Complex Numbers From A to … Z , Mathematical Olympiad in China,Mathematical Olympiad Challenges, Mathematical Olympiad

KNitviTüaCMuvijBiPBelak

- 65 - PaK1

lMhat´TI31 edayeFIVvicartamkMeN acMnYn ½ IncUrRsaybBa¢akf

2 + ckdacnwg Ct´Fm

1n1n 9A ++= E anic©cMeBa¼RKb´ n6 11

cMnYnK μCati n .

dMeNa¼Rsay RsaybBa¢ak´facMnYn 1n6A 1n

n 92 ++ += Eckdacn wg11

-ebI 0n = eKán 1192A0 =+= Bit ( Eckdac´nwg11 ) -ebI eKán 091n = 1911292A 27

1 ×==+= Bit wg ).

favaBitdltYTI KW ( Eckdacn 11

eyIg«bma p 1p1p6p 92A ++ +=

eyIgnwgRsayfavaBitdl YTI KW Ec

p 9

Eckdac´nwg ´t11 1P +

2p7p61p 92A ++

+ += kdacnwg11 2peyIgman 1 2A 7p6 ++

+ += 11p1p66

1p+ )9.29(A p62p)92(2 +++

+ −++=

9.55A64

9A64

1p

p

A

)649(A+

p1p+

1pp1

++

−=

−+=

Page 73: HU NSEN 3 STR O N G · Five Hundred Mathematical Challenges,Complex Numbers From A to … Z , Mathematical Olympiad in China,Mathematical Olympiad Challenges, Mathematical Olympiad

KNitviTüaCMuvijBiPBelak

- 66 - PaK1

edaytamkar«bma Eckdac wg ehIy .

án p 9.55A64A

pA ´n 11 1p9.55 +

Eckdac´nwg11

eKTaj 1pp1

++ −= Eckdacnwg it .

nA

11 BdUcen¼cMnYn n1n6 92 1++ += kdacnwg11 MeBa¼ Ec Canic©cb´cMnYnKt´Fm μCatin .

RK

lMhat´TI32 97831 nn

neK[ INn,617043E nn 0 ∈−+−=

ac ic©R

cUrbgHajfa nE Eckd ´nwg 187 Can Kb´ INn∈ .

dMeNa¼Rsay a Eckdac´nwg Canic b´ bgHajf ©RKnE 187 INn∈ ½

× Edl nig CacMnYnbfmrvagyfa Eckdac´nwg eyIgEtUvRsay

[eXIjfa Eckdac´ nig . ab 1nn −=

eday 187 = Kña . 1711 11 17

dUcen¼edIm,IRsa nE 187

nE 11 17

eKman ba)(ba(a 2n1nn N.)ba()b....+++−=− −−−

Page 74: HU NSEN 3 STR O N G · Five Hundred Mathematical Challenges,Complex Numbers From A to … Z , Mathematical Olympiad in China,Mathematical Olympiad Challenges, Mathematical Olympiad

KNitviTüaCMuvijBiPBelak

- 67 - PaK1

Edl 1n aaN 1n2n b.......b −−− +++= án 610709743831E nnnn

n ∈eK , INn−+−=

( ) ( )

*INq,p;)q9p8(11q99p88q)610709(p)743831(

610709743831 nnnn

∈+=+=−+−=−+−=

709743831E nnnnn

eKTaján Eckdacnwg 11 . nE

müa¨geToteKman½ INn,610 ∈−+−=

*IN;;)213(1734221

)743709()610831()743709()610831( nnnn

∈βαβ−α=β−α=β−+α−=

−+−=

Taján Eckdacnwg . ´nwg Canic©RKb´

eK nE 17

dUcen¼ n EckdacE 187 INn∈ .

Page 75: HU NSEN 3 STR O N G · Five Hundred Mathematical Challenges,Complex Numbers From A to … Z , Mathematical Olympiad in China,Mathematical Olympiad Challenges, Mathematical Olympiad

KNitviTüaCMuvijBiPBelak

- 68 - PaK1

lMhat´TI33 cUrRsaybBa¢ak´fa

1n21n

1.....3

112

1 +<+

++++

dMeN Rsay ybBa¢ak´fa ½ Rsa

1n21n

1.....3

12

11 < ++

+++ man

+

eyIg

1k21

1k1kk1kk1k 11

+=

+++>

++=−+

¦ k1k1k2

1−+<

+

ebI 1222

1:1k = −<

ebI 2332

1:2k = −<

ebI 3442

1:3k −<= ------------------------

ebI ---------

n1n1n2

1:nk −+<+

=

Page 76: HU NSEN 3 STR O N G · Five Hundred Mathematical Challenges,Complex Numbers From A to … Z , Mathematical Olympiad in China,Mathematical Olympiad Challenges, Mathematical Olympiad

KNitviTüaCMuvijBiPBelak

- 69 - PaK1

eKán 1121

−n)1n

1.........3

12

1( +<+

+++

Taj

eK 21n21n

1........3

12

1−+<

++++

1n2

11n21n

1.......32

+

1

<

−+<+

+++

dUcen¼

11+

1n21n

1.....3

112

1 +<+

++++ . lMhat´TI34 eK[ n cMnYnBitviC¢man . cUrRsaybBa¢akfa ½

n321 a.........;;a;a;a

nn321 ............aa ++ 21n3 a....a.a.anaa ≥++

dMeNa¼Rsay RsaybBa¢ak´fa ½

nn321n321 a.a.ana..........aaa ≥++++

=

a...... -cMeBa¼ eKman 2n 2121 aa2aa ≥+

Ul smm 0)aa( 221 ≥− Bit

Page 77: HU NSEN 3 STR O N G · Five Hundred Mathematical Challenges,Complex Numbers From A to … Z , Mathematical Olympiad in China,Mathematical Olympiad Challenges, Mathematical Olympiad

KNitviTüaCMuvijBiPBelak

- 70 - PaK1

-«bmafavaBitdltYTI KW ½ k

kk321k321 aaa ++ a......a.a.aka.......... ≥++ Bit

eyIgnwgRsayfavaBitdl´tYTI 1k + KW ½ 1k

1kk3211kk32 a.a......a.a.a)1k(aa..........aaa +++ +≥+++++ 1

tag *INk,0a,0x;

x)a......aaax(

)x(f k

1kk321 ∈≥>∀

+++++=

+ eyIgán ½

2

kk

xax(x)a(

)x('f1k

k2121 )a.....a....aax)(1k +++++−+++++

=

[ ]

( )2

k21k

k21

2k21

kk21

xa....aakx)a....aax(

x)a....aax(x)1k()a....aax(

−−−−++++=

++++−+++++=

ebI naM[ 0)x('f =k

a.....aax k21 +++=

cMeBa¼ k

a..... kaax 21 ++ eKán ½

+=

kk211kk21

k2

ka +1

1kk

k21

k21

ka.....aa

)1k()k

a......aa(f

a....a

)....k

a....aa(

)k

aa(f

⎟⎠⎞

⎜⎝⎛ +++

+=+++

++

+21 aaaa.... ++++++

=++

+

+

+

Page 78: HU NSEN 3 STR O N G · Five Hundred Mathematical Challenges,Complex Numbers From A to … Z , Mathematical Olympiad in China,Mathematical Olympiad Challenges, Mathematical Olympiad

KNitviTüaCMuvijBiPBelak

- 71 - PaK1

taragGefrPaBén x

)a.....aax()x(f

kk21 ++++

= x o

k..aa 21 a. k+++ ∞+

)x('f −.

+.

)x(f

m

PaBxagelIcMeBa¼ tamtaragGefr 0x ≥∀ eyIgTaján ½

kk3211k

1kk21

ka.....aaa)1k(

x)a......aa

x()x(f

⎠⎞

⎜⎝⎛ ++++

+

++++

+

+

ma ≥

=

tamkar«b kk321k321 a......a.a.aka..........aaa ≥++++

smmUl k21

ka.....++ k21 a.........a.ak

aa≥⎟

⎠⎞

⎜⎝⎛ +

eKTaj k211k

1kk21 a......aa)1k(

x)a.......aax( +

+

+≥++++

naM[ (*)xa.......a.a)1k(a.....aax 1kk21k21

++≥++++ yk CYskñúg 0ax 1k >= + ( )*

Page 79: HU NSEN 3 STR O N G · Five Hundred Mathematical Challenges,Complex Numbers From A to … Z , Mathematical Olympiad in China,Mathematical Olympiad Challenges, Mathematical Olympiad

KNitviTüaCMuvijBiPBelak

- 72 - PaK1

eKán 1k

1kk3211kk321 aaa +++ a.a......a.a.a)1k(aa.......... +++ +≥++ Bit

dUcen¼ nn321n321 .....aaa +++ a......a.a.ana..... ≥+ .

lMhat´TI35 eda¼RsayRbB&næsmIkar ½

⎩⎨

ytanxtan3

=+

=++

34ytanxtan28ytanytanxtan6xtan

3

4224

Edl 2

yx0 π<<< .

dMeNa¼Rsay eda¼RsayRbB&næsmIkar ½

⎩⎨

+ tanxtantanxt 33

=

=++

34yyan28ytanytanxtan6xtan 4224

smmUl⎩⎨⎧

=+

=++

)tanxtan4y

2(316ytanxtan4

)1(28ytanytanxtan6xtan33

4224

Page 80: HU NSEN 3 STR O N G · Five Hundred Mathematical Challenges,Complex Numbers From A to … Z , Mathematical Olympiad in China,Mathematical Olympiad Challenges, Mathematical Olympiad

KNitviTüaCMuvijBiPBelak

- 73 - PaK1

bUksmIkar nig eKán ½ )1( )2(

424 4(3162)tax(tan ==+ )31()328yn +=++ eday

2yx0 π<<< ena¼eKán ytanxtan0 <<

eKTaj )3(31ytanxtan +=+ eFIVplsg 1( nig 2() eKán ½ )

424 )31()324(1628x(tan −−− 3)ytan −=== eKTaj )4(1tanxtan −=− eRB3y a¼ ytanxtan0 <<

ar nig án bUksmIk )4( eK tan2)3( 2x = eKTaj 1xtan = naM[

4x π= .

IVplsgsmIkar nig eKán eF )3( )4( 32ytan2 = Taj eK 3ytan = naM[

3y π= .

cen¼eKánKUcemøIy dU )3

y,4

x( π=

π= .

Page 81: HU NSEN 3 STR O N G · Five Hundred Mathematical Challenges,Complex Numbers From A to … Z , Mathematical Olympiad in China,Mathematical Olympiad Challenges, Mathematical Olympiad

KNitviTüaCMuvijBiPBelak

- 74 - PaK1

lMhat´TI36 eK[ CaRCugrbs´RtIekaNmYynigtag

nRtIekaN . a

c,b,a p

Caknø¼brimaRtécUrbgHajf ⎟

⎠⎜⎛ ++≥

−+

−+

− c1

b1

a12

cp1

bp1

ap1

dMeNa¼⎝

Rsay ajfa bgH ⎟

⎠⎞

⎜⎝⎛ ++≥

−+

−+

− c1

b1

a12

cp1

bp1

ap1 MnYnBit > eKman ½

2

cMeBa¼RKb´c > 0B;0A

0A)BA( 22 ≥BB.A2 +−2 ≥++

n

=− ¦ A2 B.A4BB.A2

B.A4)BA( 2 ≥+

eKTajáBA

4B.ABA

+≥

+ ¦ )1(BA

4B11

A +≥+

yk B,apA bp −=−= án eK cbap2BA =−−=+ CYskñúg )1(

eKán )2(c4

bp11

≥−

+

dUcKñaEdreKTaj½ ap −

.

Page 82: HU NSEN 3 STR O N G · Five Hundred Mathematical Challenges,Complex Numbers From A to … Z , Mathematical Olympiad in China,Mathematical Olympiad Challenges, Mathematical Olympiad

KNitviTüaCMuvijBiPBelak

- 75 - PaK1

)3(a4

cp1

bp1

≥−

+−

nig )4(b4

ap1

cp1

≥−

+−

aB Gg:nwgGg:eKán ½ B Gg:nwgGg:eKán ½ bUkvismP )4(,)3(,)2( )4(,)3(,)2(

c4

b4

a4

cp2

bp2

ap2

++≥−

+−

+−

dUcen¼ ⎟⎠⎞

⎜⎝⎛ ++≥

−+

−+

−p1

c1

b1

a12

cp1

bp1

a .

lMhat´TI37 k¿cUrRsaybBa¢ak´fa ½

3n41n4

1n41n4

1n43n4

+−

<+−

<+− cMeBa¼RKb´

¿TajbgHajfa ½ *INn∈

x

3951n2 ×+

n43

)1n4(.......13)1n4(.......1173

+<

+×××−×××× . 1

<

dMeNa¼Rsay k¿RsaybBa¢akfa

3n41n4

1n41n4

1n43n4

+−

<+−

<+−

tamvismPaB eKGacsresr ½ GMAM −

Page 83: HU NSEN 3 STR O N G · Five Hundred Mathematical Challenges,Complex Numbers From A to … Z , Mathematical Olympiad in China,Mathematical Olympiad Challenges, Mathematical Olympiad

KNitviTüaCMuvijBiPBelak

)3n4)(1n4(1

1n41

)3n4)(1n4(1n4

)3n4)(1n4(22n8

)3n4)(1n4(2)3n4()1n4(

+−<

+

+−>+

+−>+

+−>++−

KuNGg:TaMgBIrnwg )1n4( − eyIgán ½ )1(

3n41n4

1n41n4

+−

<+−

müa¨geTot )1n4)(3n4(2)1n4()3n4( +−>++−

)1n4)(3n4(1n4

)1n4)(3n4(22n8

+−>−

+−>−

EckTaMgBIrnwg eyIgán ½ )1n4( +

)2(1n43n4

1n41n4

+−

>+−

tamTMnak´TMng nig eKTaján ½ )1( )2(

3n41n4

1n41n4

1n43n4

+−

<+−

<+− .

x¿TajbgHajfa½

3n43

)1n4(.......1395)1n4(.......1173

1n21

+<

+××××−××××

<+

- 76 - PaK1

Page 84: HU NSEN 3 STR O N G · Five Hundred Mathematical Challenges,Complex Numbers From A to … Z , Mathematical Olympiad in China,Mathematical Olympiad Challenges, Mathematical Olympiad

KNitviTüaCMuvijBiPBelak

tamsRmayxagelIeyIgman 3n41n4

1n41n4

1n43n4

+−

<+−

<+−

cMeBa¼ eyIgán ½ n;.....;3;2;1n =

3n41n4

1n41n4

1n43n4

1511

1311

139

117

97

95

73

53

51

+−

<+−

<+−

−−−−−−−−−−−−−−−−−−

<<

<<

<<

KuNvismPaBTaMgen¼Gg:nwgGg:eKán ½

3n43

1n41n4.........

1311.

97.

53

1n41

+<

+−

<+

edayeKman 1n2

14n4

11n4

1+

=+

>+

. dUcen¼

3n43

)1n4(.......1395)1n4(.......1173

1n21

+<

+××××−××××

<+

.

- 77 - PaK1

Page 85: HU NSEN 3 STR O N G · Five Hundred Mathematical Challenges,Complex Numbers From A to … Z , Mathematical Olympiad in China,Mathematical Olympiad Challenges, Mathematical Olympiad

KNitviTüaCMuvijBiPBelak

lMhat´TI38 eK[ a CabIcMnYnBitviC¢man . c;b;

cUrbgHajfa 23

bac

acb

cba

≥+

++

++

dMeNa¼Rsay bgHajfa

23

bac

acb

cba

≥+

++

++

tag ⎪⎩

⎪⎨

=+=+=+

pbanacmcb

eKán ( ) ( ) ( ) pnmbaaccb ++=+++++ naM[

2pnmcba ++

=++

eKTaj

⎪⎪⎪

⎪⎪⎪

−+=

+−=

−+=

2pnmc

2pnmb

2mpna

eKán

- 78 - PaK1

Page 86: HU NSEN 3 STR O N G · Five Hundred Mathematical Challenges,Complex Numbers From A to … Z , Mathematical Olympiad in China,Mathematical Olympiad Challenges, Mathematical Olympiad

KNitviTüaCMuvijBiPBelak

- 79 - PaK1

p2pnm

n2pnm

m2mpn

bac

acb

cba −+

++−

+−+

=+

++

++

⎥⎦

⎤⎢⎣

⎡−⎟

⎞⎜⎝

⎛++⎟

⎞⎜⎝

⎛+⎟

⎞⎜⎝⎛ ++

++

+ nm

mn

acb

cba

+⎠

=+

3np

pn

pm

mp

21

bac

amvismPaB eKán ½ t GMAM −

2np

pn;2

pm

mp;2

nm

mn+ ≥+≥+≥

eKán ( )23322

21

bac

acb

ca

=−+≥ 2b

++

++

++

dUcen¼ 23

bac

acb

cba

≥+

++

+ . +

lMhat´TI39 k¿KNna

n)1n(3.21

2.11

−++ Edl 2n > 1....

4.31

++ x¿edayeRbIvismPaB GMAM − én n( )1− cMnYnxageRkam ½

n)1n(1;

4.33.22.11;1;1

− cUrbgHajfa 2n )!n(n < .

dMeNa¼Rsay k¿KNna ∑

=⎥⎦

⎤⎢⎣

⎡−

=−

++++n

2k k)1k(1

n)1n(1....

4.31

3.21

2.11

Page 87: HU NSEN 3 STR O N G · Five Hundred Mathematical Challenges,Complex Numbers From A to … Z , Mathematical Olympiad in China,Mathematical Olympiad Challenges, Mathematical Olympiad

KNitviTüaCMuvijBiPBelak

- 80 - PaK1

eKman k1

1k1

k)1k()1k(k

k)1k(1

−−

=−

−−=

eKán n11

k1

1k1

k)1k(1 n

2k

n

k∑=2

−=⎟⎠⎞

⎜⎝⎛ −

−=⎥

⎤⎢⎣

⎡−

∑=

dUcen¼ n11

n)1n(1...

2.11+ .

.31

3.21

−=−

+++4

. n(n <

tamvismPaB ½ x¿ bgHajfa 2)!n

GMAM −

nn321n321 a......a.a.ana.....aaa ++ ≥++

eKán ½ 0a; k ≥∀

)1n(

n)1n(1......

3.21.

2.11)1n(

n)1n(1....

3.21

2.11

−−>

−+++

2n2

1n

)1n(2

)1n(

)1n(

)!n(n1

>⎟⎠⎞

⎜⎝⎛ n

)!n(n

)!n(n

n1

!)1n(!n1)1n(

n1n

!)1n(!n1)1n(

n11

<⇒

>

−−>

−−>−

dUcen¼ 2n )!n(n < .

Page 88: HU NSEN 3 STR O N G · Five Hundred Mathematical Challenges,Complex Numbers From A to … Z , Mathematical Olympiad in China,Mathematical Olympiad Challenges, Mathematical Olympiad

KNitviTüaCMuvijBiPBelak

- 81 - PaK1

lMhat´TI40 eK[ ;b;a c CabIcMnYnBitxusBIsUnü .

UrbgHajfa cac

cb

ba

ac

cb

ba 222

++≥⎟⎠⎞

⎜⎝⎛+⎟

⎠⎞

⎜⎝⎛+⎟

⎠⎞

⎜⎝

dMeNa¼Rsay

bgHajfa ac

cb

ba

ac

ba

+≥⎟⎠⎞

⎜⎝⎛+⎜

⎝⎛

cb 222

+⎟⎠⎞

⎜⎝⎛+⎟

⎠⎞

tamvismPaB eKman ½ GMAM −

3b

⎜⎝

+⎟⎠

⎜⎝

ac.

cb.

ba3

ac

cba

32

2

2

2

2

2222

=≥⎟⎠⎞

⎜⎝⎛+⎟

⎠⎞⎛⎞⎛

eKTaj )1(3ac

cb

ba 222

≥⎟⎠⎞

⎜⎝⎛+⎟

⎠⎞

⎜⎝⎛+⎟

⎠⎞

⎜⎝⎛

müa¨geToteKman

⎪⎪⎪⎪

⎪⎪⎪⎪

≥+⎟⎠⎞

⎜⎝⎛

≥+⎟⎠⎞

⎜⎝⎛

≥+⎟⎠⎞

⎜⎝⎛

ac21

ac

cb21

cb

ba21

ba

2

2

2

eKTaján )2(ac

cb

ba23

ac

cb

⎜⎝⎛ +≥+⎟

⎠⎞

⎜⎝⎛+⎜

⎝⎛+⎜

⎝⎛

ba 222

⎟⎠⎞+⎟

⎠⎞

⎟⎠⎞

Page 89: HU NSEN 3 STR O N G · Five Hundred Mathematical Challenges,Complex Numbers From A to … Z , Mathematical Olympiad in China,Mathematical Olympiad Challenges, Mathematical Olympiad

KNitviTüaCMuvijBiPBelak

- 82 - PaK1

bUkvismPaB nig Gg: nig Gg: eKán ½ )1( )2(

⎟⎠⎞

⎜⎝⎛ +++≥⎥

⎤⎢⎣

⎡⎟⎠⎞

⎜⎝⎛+⎟

⎠⎞

⎜⎝⎛+⎟

⎠⎞

⎜⎝⎛+

ac

cb

ba23

ac

cb

ba23

222

⎟⎠⎣ ⎝ b⎞

⎜⎝⎛ ++≥⎥

⎤⎢⎡

⎟⎠⎞

⎜⎝⎛+⎟

⎠⎞

⎜⎝⎛+⎟

⎠⎞

⎜⎛

ac

cb

ba2

ac

cba2

222

Ucen¼ dac

cb

ba

ac

cb

b⎝a 222

++≥⎟⎠⎞

⎜⎝⎛+⎟

⎠⎞

⎜⎝⎛+⎟

⎠⎞

⎜⎛ .

lMhat´TI41 eK[ ;y;x z CabIcMnYnBitviC¢manminsUnü . UrbgHajfa c 222222 zyx

9z1

y1

x

1++

≥++

dMeNa¼Rsay bgHajfa 222222 yxzyx +

≥++z

9111+

vismPaBen¼GacsresreTACa ½

9z

zyxy

zyxx

zyx2

222

2

222

2

222

≥++

+++

+++

6zy

yz

zx

xz

yx

xy

9zy

zx1

yz

yx1

xz

xy1

2

2

2

2

2

2

2

2

2

2

2

2

2

2

2

2

2

2

2

2

2

2

2

2

≥+++++

≥++++++++

Page 90: HU NSEN 3 STR O N G · Five Hundred Mathematical Challenges,Complex Numbers From A to … Z , Mathematical Olympiad in China,Mathematical Olympiad Challenges, Mathematical Olympiad

KNitviTüaCMuvijBiPBelak

eday 2yx.

xy2

yx

xy

2

2

2

2

2

2

2

2

=≥+

2zx.

xz2

zx

xz

2

2

2

2

2

2

2

2

=≥+ nig 2yz.

zy2

yz

zy

2

2

2

2

2

2

2

2

=≥+

eKán 6222zy

yz

zx

xz

yx

xy

2

2

2

2

2

2

2

2

2

2

2

2

=++≥+++++ Bit

dUcen¼ 222222 zyx9

z1

y1

x1

++≥++ .

lMhat´TI42 eKman xlog1

1

aay −= nig ylog11

aaz −=

cUrRsayfa zlog11

aax −= .

dMeNa¼Rsay Rsayfa zlog1

1

aax −=

eKman xlog11

aay −= naM[ xlog1

1yloga

a −=

eKTaj )1(ylog

11xloga

a −=

- 83 - PaK1

Page 91: HU NSEN 3 STR O N G · Five Hundred Mathematical Challenges,Complex Numbers From A to … Z , Mathematical Olympiad in China,Mathematical Olympiad Challenges, Mathematical Olympiad

KNitviTüaCMuvijBiPBelak

dUcKñaEdr ylog11

aaz −= naM[ ylog1

1zloga

a −=

eKTaj )2(zlog

11yloga

a −=

ykTMnak´TMng eTACYskñúg eKán ½ )2( )1(

zlog11

11xlog

a

a

−−=

1)z(logzlog

1xloga

aa −

−=

zlog11xlog

1)z(logzlog1zlog

xlog

aa

a

aaa

−=

−−−

=

eKTaján zlog11

aax −= .

- 84 - PaK1

Page 92: HU NSEN 3 STR O N G · Five Hundred Mathematical Challenges,Complex Numbers From A to … Z , Mathematical Olympiad in China,Mathematical Olympiad Challenges, Mathematical Olympiad

KNitviTüaCMuvijBiPBelak

- 85 - PaK1

lMhat´TI43

eKBinitüsIVúténcMnYnBit ( kMnt´eday )un⎪

⎪⎨

−−

=

=

+ 1u24u5

u

4u

n

n1n

1

cUrKNna u CaGnuKmn_én . n n

dMeNa¼Rsay KNna u CaGnuKmn_én ½ n n

smIkarsMKal´énsIVút 1u24u5

un

n1n −

−=+ KW

1r24r5r

−−

=

¦ man¦s 04r6r2 2 =+− 2acr;1r 21 ===

tagsIVút 2u1uv

n

nn −

−=

eKán nn

n

n

n

n

n

1n

1n1n v3)

2u1u

(32

1u24u5

11u24u5

2u1u

v =−−

=−

−−

−−−

=−−

=+

++

TMnak´TMngen¼bBa¢ak´fa CasIVútFrNImaRtmanersug )v( n

nigtY 3q =23

2u1u

v1

11 =

−−

= .

Page 93: HU NSEN 3 STR O N G · Five Hundred Mathematical Challenges,Complex Numbers From A to … Z , Mathematical Olympiad in China,Mathematical Olympiad Challenges, Mathematical Olympiad

KNitviTüaCMuvijBiPBelak

tamrUbmnþ 233

23qvv

n1n1n

1n =×=×= −−

eday 2u1uv

n

nn −

−= eKTaj

123

131v1v2u n

n

n

nn

−=

−−

=

dUcen¼ 23

)13(2v n

n

n −−

= . lMhat´TI44

eKBinitüsIVúténcMnYnBit kMnt´eday )u( n⎪⎩

⎪⎨

−−

=

=

+ 1u4u3

u

3u

n

n1n

1

cUrKNna CaGnuKmn_én . nu n

dMeNa¼Rsay KNna CaGnuKmn_én ½ nu n

smIkarsMKal´énsIVút 1u4u3

un

n1n −

−=+ KW

1r4r3r

−−

=

¦ 0)2r(4r4r 22 =−=+− eKTaj¦s 2rr 21 == . tagsIVút

2u1v

nn −=

- 86 - PaK1

Page 94: HU NSEN 3 STR O N G · Five Hundred Mathematical Challenges,Complex Numbers From A to … Z , Mathematical Olympiad in China,Mathematical Olympiad Challenges, Mathematical Olympiad

KNitviTüaCMuvijBiPBelak

- 87 - PaK1

eKán 2u

1v1n

1n −=

++ eday

1u4u3

un

n1n −

−=+

eKán 2u1u

2u24u31u

vn

n

nn

n1n −

−=

+−−−

=+

eKman 12u

12u1u

vvnn

nn1n =

−−

−−

=−+ efr

naM[ CasIVútnBVnþmanplsgrYm )v( n 1d = nigtY 1

231

2u1v

11 =

−=

−= .

tamrUbmnþ n1n1d)1n(vv 1n =−+=−+= eday

2u1v

nn −= eKTaj

n1n2

n12

v12u

nn

+=+=+=

dUcen¼ n

1n2un+

= .

Page 95: HU NSEN 3 STR O N G · Five Hundred Mathematical Challenges,Complex Numbers From A to … Z , Mathematical Olympiad in China,Mathematical Olympiad Challenges, Mathematical Olympiad

KNitviTüaCMuvijBiPBelak

- 88 - PaK1

lMhat´TI45 eKBinitüsIVúténcMnYnBit kMnt´eday ½ )u( n

⎩⎨⎧ = 4u1

∈=+ *INn,uu n1n

cUrKNna CaGnuKmn_é .

MeNa¼

nu n n

d Rsay aGnuKmn_én KNna nu C n

eKman n1 uu = n+

eKán n1n ulnln21u =+

gen¼bB¢ak´fa (l n CasIVútFrNImaRtmanersug TMnak´TMn un )21q =

tamrUbmnþ ( ) 1n2

1

1n1

n 4ln4ln1ulnuln −=== − n1 2

q× −

dUcen¼ ( ) 1n2

1

n 4u −= .

Page 96: HU NSEN 3 STR O N G · Five Hundred Mathematical Challenges,Complex Numbers From A to … Z , Mathematical Olympiad in China,Mathematical Olympiad Challenges, Mathematical Olympiad

KNitviTüaCMuvijBiPBelak

- 89 - PaK1

lMhat´TI46 eK[sIVúténcMnYnBit kMnt´eday ½ )u( n

5u0 = nig 5nu21u n ++ 1n =+ ( INn∈ )

cUrKN CaGnuKmn_én

MeNa¼

na nu énsIVút n . )u( n

d Rsay sIVút CaGnuKmn_én

u

KNna nu én )u( n n

eyIgtag bvnn an ++= Edl IRb,a ∈ eyIgán baanvu 1n1n +++= ++ eday 5nu

21u n1n+ ++=

eKán 5n)banvn ++v(21baan n1 ++=++++

(*))b21a5(n)1a

21(v

21v n1n ⎥⎦

⎤⎢⎣⎡ −−++−+=+

ebI ⎪⎪⎩

⎪⎪⎨

⎧ −+− 01a1

=−− 0b21a5

2 naM[ 3b;2a ==

TMnak´TMng eTACa ( )* n1n v21v =+ naM[ CasIVútFrNImaRt. )v( n

manersug 21

= . q

Page 97: HU NSEN 3 STR O N G · Five Hundred Mathematical Challenges,Complex Numbers From A to … Z , Mathematical Olympiad in China,Mathematical Olympiad Challenges, Mathematical Olympiad

KNitviTüaCMuvijBiPBelak

- 90 - PaK1

tamrUbmnþ 0 qv × nn v= eday 6n2vbanvu nnn ++=++= eKán 0cMeBa¼ n = 6v00 u +=

u0naM[ 56v 0 16 −=−=−= eKán nnn 2

1211v −=×−= .

dUcen¼ 6n221u nn ++−= .

lMhat´TI47 edayeRbIGnumanrYmKNitviTüacUrKNnalImItxageRkam ½

2x2x22.....222

limLn 2x −−++++++

= →

( man n ra¨DIkal´ ) .

MeNa¼

d Rsay ageRkam ½ KNnalImItx

eyIgman 2x

2x22.....222limLn 2x −

−+++++→

+=

eyIgán

Page 98: HU NSEN 3 STR O N G · Five Hundred Mathematical Challenges,Complex Numbers From A to … Z , Mathematical Olympiad in China,Mathematical Olympiad Challenges, Mathematical Olympiad

KNitviTüaCMuvijBiPBelak

- 91 - PaK1

41

2x21lim

)2x2)(2x(4)x2(lim

2x2x2limL

2x1 = → 2x2x=

++=

++−−+

=−

−+→→

212x2x2

2x2x2

41L

41

2x2x2lim

2x221limL

)2x22)(2x(2x2lim

2x2x22limL

==−−+

×+++

=

+++−

−+=

−−++

=

→→

→→

eyIgsnμtfavaBitdllMdab´TI KW k kk 41L =

´TeyIgnwgRsayfavaBitdl´lMdab I )1k( + KW 1k1k 41L ++ =

eyIgman 2x

2x2.....222 2limL

2x1k −−++

→+ knigPaKEbgnwg

++++=

KuNPaKy2x2..2222)x(M ++++= . +++

eKán )x(M).2x(

4x2....2222limL2x1k −

−++++++=

→+

Bit1kkkk1k

2x2x1k

41

41

41L

41L

)2(M1L

2x2x2....222lim

)x(M1limL

++

→→+

=×==×=

−−+++++

×=

dUcen¼ n2xn 41

2x2x22.....222

limL =−

−++++++=

Page 99: HU NSEN 3 STR O N G · Five Hundred Mathematical Challenges,Complex Numbers From A to … Z , Mathematical Olympiad in China,Mathematical Olympiad Challenges, Mathematical Olympiad

KNitviTüaCMuvijBiPBelak

- 92 - PaK1

lMhat´TI48 eK[sIVúténcMnYnBit kMnt´elI eday ½ )U( n IN

⎪⎩

⎪⎨⎧ = 2U0

∈+=+ INn,U2U n1n

¿cUrKNna CaGnuKmn_én . KuN 1 U....

k nnU

x¿KNnapl 20n UUUP n××××= .

eNa¼

dM Rsay aGnuKmn_én ½

KNna nU C n

eyIgman4

cos220U π==

8

cos24

cos22U2U 01π

+=+=

itdl´tYTI KW «bmafavaB p 2pp 2cos2U +

π=

I eyIgnwgRsayfavaBitdl´tYT )1p( + KW 3p1p+ 2cos2U +

π= Bit

eyIgman p1p U2U +=+ Et r«bmtamka a 2pp 2cos2U +

π=

eyIgán 3p3p

22p cos4U ++

π=

π 1p 2cos2

22cos22 ++

π=+= Bit

Page 100: HU NSEN 3 STR O N G · Five Hundred Mathematical Challenges,Complex Numbers From A to … Z , Mathematical Olympiad in China,Mathematical Olympiad Challenges, Mathematical Olympiad

KNitviTüaCMuvijBiPBelak

- 93 - PaK1

dUcen¼ 2nn 2cos2U +

π= .

aplKuN 0n U....UUPx¿ KNn 1U n2 ××××= naM[ tamrUbmnþ 2a2sin = acosasin

ana2n

sisiacos2 =

2n2n

n 1k2sin +

0k

n

0k2k

n

0k2kkn

2sin

1

2sin

2sin

)

2sin

()2

cos2()U(P

++

= =+

=+

π=

π

π

=

π

π

==∏ ∏∏

dUcen¼ 2n

n

2sin

1P+

π= .

Mhat´TI49 l

MnYnBit kMnt´elI eday ½ eK[sIVúténc )U( n IN

22U0 = nig INn,

211

U2

n1n ∈∀

−−+

U=

KNna nuKmn_én .

n CaGU n

Page 101: HU NSEN 3 STR O N G · Five Hundred Mathematical Challenges,Complex Numbers From A to … Z , Mathematical Olympiad in China,Mathematical Olympiad Challenges, Mathematical Olympiad

KNitviTüaCMuvijBiPBelak

- 94 - PaK1

dMeNa¼Rsay KNna CaGnuKmn_én ½ n

U n

eyIgman4

sin20 ==2 π U

8

sin2

4sin11

2U11

U1 =

22

0 π=

π−−

=−−

favaBitdl´tYTI KW «bma p 2pp 2sinU +

π=

I eyIgnwgRsayfavaBitdl´tYT )1p( + KW 3p1pU + 2sin += Bit π

eyIgman 2

U11U 1p

−−=+

2p kar«bma 2+pp 2

sinU π=

eyIgán 2

2U2p

2+

sin111p+

π−−

=

3p

3+=

p2

2p

2sin

22

sin2

22

cos1+

+ π=

π

=

π−

Bit

dUcen¼ 2nn 2sinU +

π= .

Page 102: HU NSEN 3 STR O N G · Five Hundred Mathematical Challenges,Complex Numbers From A to … Z , Mathematical Olympiad in China,Mathematical Olympiad Challenges, Mathematical Olympiad

KNitviTüaCMuvijBiPBelak

- 95 - PaK1

lMhat´TI50 eK[sIVúténcMnYnBit )u( n kMnt´eday½

⎧ = au0

⎩⎨

>∈−=+ 2a,IN,2uu 2n1n ∀n

rRsaybBa¢ak´fa cUnn 2

22

2

n 24aa

24aau ⎟

⎟⎠

⎞⎜⎜⎝

⎛ −−+⎟

⎟⎠

⎞⎜⎜⎝

⎛ −+=

dMeNa¼Rsay

RsaybBa¢ak´fa nn 2

22

2

n 24aa

24aau ⎟

⎟⎠

⎞⎜⎜⎝

⎛ −−+⎟

⎟⎠

⎞⎜⎜⎝

⎛ −+=

ebI 0n = eKán a2

4aa2

4aau22

0 =−−

+−+

= Bit

avaBit KW

«bm dltYTI kkk 2

22

2

k 24⎟⎟⎠

aa2

4aau⎞

⎜⎜⎝

⎛ −−+⎟

⎟⎠

⎞⎜⎜⎝

⎛ −+=

itdl´tYTeyIgnwgRsayfavaB I 1k + KW 1k1k 2

22

2

1k 22u +⎟

⎟⎠

⎜⎜⎝

=+

4aa4aa++

⎟⎟⎠

⎞⎜⎜⎝

⎛ −−⎞⎛ −+

eyIgman 2uu 2k1k −=+

Page 103: HU NSEN 3 STR O N G · Five Hundred Mathematical Challenges,Complex Numbers From A to … Z , Mathematical Olympiad in China,Mathematical Olympiad Challenges, Mathematical Olympiad

KNitviTüaCMuvijBiPBelak

- 96 - PaK1

Ettamkar«bma kk 2

22

2

k 24aa

24aau ⎟

⎟⎠

⎞⎜⎜⎝

⎛ −−+⎟

⎟⎠

⎞⎜⎜⎝

⎛ −+=

eyIgán 22

4aa2

4aau

22

22

2

1k

kk

−⎥⎥

⎢⎢

⎟⎟⎠

⎞⎜⎜⎝

⎛ −−+⎟

⎟⎠

⎞⎜⎜⎝

⎛ −+=+

24

4aa22

4aa2

4aau22

22

22

1k

1k1k

−+−

×+⎟⎟⎠

⎞⎜⎜⎝

⎛ −−+⎟

⎟⎠

⎞⎜⎜⎝

⎛ −+=

++

+

1k1k 2

22

2

1k 24aa

24aau

++

⎟⎟⎠

⎞⎜⎜⎝

⎛ −−+⎟

⎟⎠

⎞⎜⎜⎝

⎛ −+=+ Bit

dUcen¼ nn 2

22

2

n 24aa

24aau ⎟

⎟⎠

⎞⎜⎜⎝

⎛ −−+⎟

⎟⎠

⎞⎜⎜⎝

⎛ −+= .

Page 104: HU NSEN 3 STR O N G · Five Hundred Mathematical Challenges,Complex Numbers From A to … Z , Mathematical Olympiad in China,Mathematical Olympiad Challenges, Mathematical Olympiad

KNitviTüaCMuvijBiPBelak

- 97 - PaK1

lMhat´TI51 k¿cUrkMnt´elxénGBaØat éncMnYn d,c,b,a abcd ebIeKdwgfa ½

dcba9abcd =× x¿cMeBa¼témø ,b,a EdlánrkeXIjxagelIcUrbBa¢ak´fa d,c

cMnYn abcd nig dcba suTæEtCakaer®ákd .

dMeNa¼Rsay k¿kMntelxénGBaØat ½ d,c,b,a

eKman )1(dcba9abcd =× k´TMng eKTajánté EtmYyKtKW . tamTMna mø )1( a 1a =

cMeBa¼ 1a = eKán )2(1dcb9bcd1 =× tamTMna g )2( eKk´TMn Taján 9d = eRBa¼ =×

es d 819

manelxxagcug μI 1 . cMeBa¼ 9d = eKán )3(1cb999bc1 × =

k´TMn Taján tamTMna g )3( eK 0b = inG T ) ( eRBa¼ 9b× m acmanRtaTuke

Page 105: HU NSEN 3 STR O N G · Five Hundred Mathematical Challenges,Complex Numbers From A to … Z , Mathematical Olympiad in China,Mathematical Olympiad Challenges, Mathematical Olympiad

KNitviTüaCMuvijBiPBelak

- 98 - PaK1

cMeBa¼ b eKán 0= )3(01c999c10 =× k´TMn Taján tamTMna g )3( eK 8c = eRBa¼ 72989c =×=×

Efm 8 [elxxagcugesμI 0 . cMeBa¼ 8c = eKán 91089 9801=× .

adUcen¼ 0b,1 9d,8c, === .= ´facMnYn x¿bBa¢ak abcd nig dcba suT aer®ákd ½

0

æEtCakcMeBa¼ 98c,b,1a d, ==== eKán ½

2331089abcd = = nig 2991980dcba == suTæEtCakaer®ákd

.

Page 106: HU NSEN 3 STR O N G · Five Hundred Mathematical Challenges,Complex Numbers From A to … Z , Mathematical Olympiad in China,Mathematical Olympiad Challenges, Mathematical Olympiad

KNitviTüaCMuvijBiPBelak

- 99 - PaK1

lMhat´TI52 sn μtfa HUNSEN CacMnYnmYymanelxRáMmYyxÞg´Edl

eBlEdleKKuNcMnYn YlTæplCacMnYnmanelx N,E,S,N,U,H Caelx . en¼nwg 3 eKTT

RáMmYyxÞg´KW STRONG Edl G,N,O,R,T,S Caelx cUrrkcMnYn HU ig NSEN n STRONG

= 8NcMeBa¼ N = nigcMeBa¼ S,3S,6 9== . usKñatageday ). (Gkßrx elxxusKña

STRONG3

HUNSEN×

dMeNa¼Rsay rkcMnYn HUNSEN nig STRONG

Page 107: HU NSEN 3 STR O N G · Five Hundred Mathematical Challenges,Complex Numbers From A to … Z , Mathematical Olympiad in China,Mathematical Olympiad Challenges, Mathematical Olympiad

KNitviTüaCMuvijBiPBelak

- 100 - PaK1

eKman ½

STRONG

3HU×

NSEN

k¿cMeBa¼ 3S,6N ==

G6TRO33

6E63HU×

tamtaragRbmaNviFIKuNen¼bBa¢ak´fa 1H =

cMeBa¼ 1H = taragviFIKuNeTACa ½

G6TRO3

36E36U1

×

tamtaragviFIKuNen¼eKTaján 8G = eRBa¼

eTACa ½

36×

manelxagcugesμI 8 . cMeBa¼ 8G = taragviFIKuN

68TRO3

36E63U1

×

tamtaragviFIKuNen¼eKTaján 5E = eRBa¼ Efm 3E× 1

Page 108: HU NSEN 3 STR O N G · Five Hundred Mathematical Challenges,Complex Numbers From A to … Z , Mathematical Olympiad in China,Mathematical Olympiad Challenges, Mathematical Olympiad

KNitviTüaCMuvijBiPBelak

- 101 - PaK1

manelxcugeRkayesμInwg 6 . cMeBa¼ 5E = taragviFIKuN TACa e ½

68TRO3

363U1

×56

tamtaragviFIKuNen¼eKTaján 0O = eRBa¼ Efm

½

33× 1

manelxcugeRkayesμInwg 0 . cMeBa¼ 0O = taragviFIKuNeTACa

068TR33

6U1 356×

tamtaragviFIKuNen¼eKTaján 9R = eRBa¼ Efm 36× 1

manelxcugeRkay esμInwg 9 . cMeBa¼ 9R = taragviFIKuNeTACa ½

9068T33

6U1 356×

Page 109: HU NSEN 3 STR O N G · Five Hundred Mathematical Challenges,Complex Numbers From A to … Z , Mathematical Olympiad in China,Mathematical Olympiad Challenges, Mathematical Olympiad

KNitviTüaCMuvijBiPBelak

- 102 - PaK1

tamtaragen¼bBa¢ak´fa minGacFMCag eTehIyedayGkßr xusKñatagedayelxxusKñaena¼eKTaján

U 2

2U = EtmYyKt BIeRBa¼ ehIy minGacyktémø ¦ 1H,0O == U 0 1

dEdleTotáneT . cMeBa¼ 2U = taragviFIKuNeTACa ½

90T3 68

56

tamtaragviFIKuNen¼eKTaján

3× 1263

7T = BIeRBa¼fa ½

379068 3

126356×

edayeRbobeFobtaragviFIKuN ½

STRONG nwg 3

HUNSEN×

379068

3126356×

Page 110: HU NSEN 3 STR O N G · Five Hundred Mathematical Challenges,Complex Numbers From A to … Z , Mathematical Olympiad in China,Mathematical Olympiad Challenges, Mathematical Olympiad

KNitviTüaCMuvijBiPBelak

- 103 - PaK1

eKTaján ½ 126356 nigHUNSEN = 379068STRONG =

CacMnYnEdlRtUvrk . ¿cMeBa¼ x 9S,8N ==

tamrebobdUcxagelIeKán ½ eyIgeda¼Rsay

9567843×

318928

edayeRbobeFobtaragviFIKuN ½

STRONG3

HUNSEN× nwg

9567843

318928×

eKTaján ½ 318928HUNSEN = nig 956784STRONG =

CacMnYnEdlRtUvr

k .

Page 111: HU NSEN 3 STR O N G · Five Hundred Mathematical Challenges,Complex Numbers From A to … Z , Mathematical Olympiad in China,Mathematical Olympiad Challenges, Mathematical Olympiad

KNitviTüaCMuvijBiPBelak

- 104 - PaK1

lMhat´TI53 cUrbMeBjRbmaNviFIbUkxageRkam ½

YENOM

EROM

dMeNa¼

DNES+

(GkßrxusKñatagedayelxxusKña )

Rsay bMeBjRbmaN UkxageRkam ½ viFIb

YENOMEROM

+DES

N

cemøIyrbs´taragviFIbUken¼KW ½

256015801

7659+

.

( sisßeda¼RsayedayxøÜnÉg )

Page 112: HU NSEN 3 STR O N G · Five Hundred Mathematical Challenges,Complex Numbers From A to … Z , Mathematical Olympiad in China,Mathematical Olympiad Challenges, Mathematical Olympiad

KNitviTüaCMuvijBiPBelak

- 105 - PaK1

lMhat´TI54 eK[ACacMnYnmYymanelxRáMxÞg´EdlelxxÞg´vaerobtamlMdab´

x;1x;x +x;2x;1 ++ ehIy CacMnYnmYymanelx ´

B

RáMmYyxÞg´EdlelxxÞg´vaerobtamlMdabx;1x;1x;2x;1x;x −++ .

Ha

dMeNa¼

+

cUrbg jfaebI A CakaerRákdena¼ B k¾Cakaer®ákdEdr .

Rsay

eyIgmanelxTaMgRáMxÞg´én A erobtamlMdab´ karbgHaj ½

x;1x2x;1x;x +++ naM[ 92x1x0 ≤+<; x +<< ¦ 7x0 ≤< .

a¼ -ebI 1x = en 211112321A == kd .

-ebI ena¼ A = mnCakaer®ákd . I e minEmnCakaer®ákd .

= minEmnCakaer®ákd .

-ebI 2x = ena¼ 23432A = minEmnCakaer®á3x = 34543 minE

-eb na¼ 4x = 45654A =

-ebI x ena¼ 5 56765A =

Page 113: HU NSEN 3 STR O N G · Five Hundred Mathematical Challenges,Complex Numbers From A to … Z , Mathematical Olympiad in China,Mathematical Olympiad Challenges, Mathematical Olympiad

KNitviTüaCMuvijBiPBelak

- 106 - PaK1

-ebI ena¼ minEmnCakaer®ákd . 6x = 67876A =

-ebI ena¼ minEmnCakaer®ákd . 7x = 78987A =

srubmkeKán x øEdl[ A Cakaer®ákd 1= Catém . ´vaerobtaml

;1

eday elxxÞg Mdab´ ½ B

x;1;2xx;x x;1x −++

MeBa¼ eKán +

ena¼c 1x = 2351123201B == Cakaer®ákd . ¾Cakaer®ákdEdr . dUcen¼ ebI A CakaerRákdena¼ B k

lMhat´TI55 eK[cMnYn

)elxn2(

11.....11A 11= nig )elxn(

444.....444B =

cUrbgHajfa 1B ++ CakaerRákd . A

dMeNa¼Rsay bgHajfa A 1B ++ CakaerRákd ½ eyIgman

9110111A

n2 −=.....111

n2(

=)elx

Page 114: HU NSEN 3 STR O N G · Five Hundred Mathematical Challenges,Complex Numbers From A to … Z , Mathematical Olympiad in China,Mathematical Olympiad Challenges, Mathematical Olympiad

KNitviTüaCMuvijBiPBelak

- 107 - PaK1

9

)110(4444.....444n

n(

−=B =

)e

lx

eyIgán 19

)19

1BAn

+−

+=++

10(4110 n2 −

2n

nn2

nn2

32101BA

9410.4101BA

99410.41101BA

⎟⎟⎠

⎞⎜⎜⎝

⎛ +=++

++=++

+−+−=++

dUcen¼ CakaerRákd . 1BA ++

lMhat´TI56 cMnYnmYymanelxbYnxÞg´EdlelxxÞg´vaerobtamlMdab´

b;b;a;a . IeKdwgfavaCakaerRákd . rkcMnYnena¼eb

dMeNa¼Rsay rkcMnYnEdlCakaerRákd ½ tag N CacMnYnEdlRtUvrk

Page 115: HU NSEN 3 STR O N G · Five Hundred Mathematical Challenges,Complex Numbers From A to … Z , Mathematical Olympiad in China,Mathematical Olympiad Challenges, Mathematical Olympiad

KNitviTüaCMuvijBiPBelak

- 108 - PaK1

eyIgán bb10a100a1000aabbN +++==

++=

)ba100(11Nb11a1100N

+= [ ] )1()ba(a9911+=

eday 9a0 ≤< nig b0 9≤≤ ena¼ 18ba0 ≤+< Mng edIm,I[ GacCakaerRákdlu¼RtaEt huKuNén ehIy

tamTMnak´T )1( N

ba + CaB 11 18ba0 ≤+< ena¼eKRtUv[ Etm .

Ga11ba =+ YyKt

TMnak´TMng )1( csresr ( ) 1a9(1111a9911 2 )N +=+= ehtue k a n¼ N Cakaer®á dkalN 1a9 + Cakeday ≤

aerRákd . 9a ≤ naM[ 101 811a9 ≤+≤ eKTaj ½

)

= (K ñúg )

161a9 =+ (Kμan¦skñúg IN

1a9 =+ (K μan¦skñúg IN ) 25

1a9 + μan¦sk36 IN

(K μan¦skñúg ) 491a9 =+ IN

naM[ 7a641a9 =+ = e Iy h 4711b =−=

cen¼cMnYnEd KW 4

(K μan¦skñúg I ) . 811a9 =+ N

dU lRtUvkMnt´ena¼ 774 288= .

Page 116: HU NSEN 3 STR O N G · Five Hundred Mathematical Challenges,Complex Numbers From A to … Z , Mathematical Olympiad in China,Mathematical Olympiad Challenges, Mathematical Olympiad

KNitviTüaCMuvijBiPBelak

- 109 - PaK1

lMhat´TI57 eK 0889333,1089 2man 33 22 = 111088893333,11==

. agelIcUrrkrUbmnþTUeTA nigRsaybBa¢ak´rUbmnþen¼pg

1111088889333332 =

BI«TahrN_x

dMeNa¼Rsay eKman 111088893333,110889333,108933 222 === .

IgGacbeg;ItrUbmnþTUeTAdUcxageRkam ½ ...

)1)1n()

2

−−

=

1111088889333332 =

tamlMnaMen¼ey9888.....8880111.....111333..333

n(n(

.

eyIgtag karRsaybBa¢ak´rUbmnþ ½

9888.........8880111..........111A)1n()1n( −−

=

Page 117: HU NSEN 3 STR O N G · Five Hundred Mathematical Challenges,Complex Numbers From A to … Z , Mathematical Olympiad in China,Mathematical Olympiad Challenges, Mathematical Olympiad

KNitviTüaCMuvijBiPBelak

)n(

2

2n2nnn2

n1nn2

1n1n1n

)1n(

1n

)1n(

333............333

3110

9)110(

9110.210

9818010.81010

910)110(9810)110(

91

910.888.........88810111.........111

=

⎟⎟⎠

⎞⎜⎜⎝

⎛ −=

−=

+−=

+−+−=

+−+−=

++×=

+

−+−

+

dUcen¼eKánrUbmnþ ½ 9888.....8880111.....111333.....333

)1n()1n()n(

2

−−

= .

- 110 - PaK1

Page 118: HU NSEN 3 STR O N G · Five Hundred Mathematical Challenges,Complex Numbers From A to … Z , Mathematical Olympiad in China,Mathematical Olympiad Challenges, Mathematical Olympiad

KNitviTüaCMuvijBiPBelak

lMhat´TI58 eKman 999800019999,998001999,980199 222 === . 9999800001999992 =

BI«TahrN_xagelIcUrrkrUbmnþTUeTAnigRsaybBa¢ak´rUbmnþen¼pg

dMeNa¼Rsay tambMrab´eKman ½

999800019999998001999

980199

2

2

2

=

=

=

9999800001999992 = tamlMnaMen¼eyIgGacbeg;ItrUbmnþTUeTAdUcxageRkam ½

1000.....0008999.....999999.....999)1n()1n()n(

2

−−

= .

karRsaybBa¢ak´rUbmnþ ½ eyIgtag 1000.........0008999..........999A

)1n()1n( −−

=

- 111 - PaK1

Page 119: HU NSEN 3 STR O N G · Five Hundred Mathematical Challenges,Complex Numbers From A to … Z , Mathematical Olympiad in China,Mathematical Olympiad Challenges, Mathematical Olympiad

KNitviTüaCMuvijBiPBelak

)n(

22n

nn2

n1nn2

n1n1n

n1n

)1n(

999............999)110(110.210

110.81010110.810)110(

110.810999.........999

=−=

+−=

++−=

++−=

++×=

+

+−

+

dUcen¼eKánrUbmnþ½

1000.....0008999.....999999.....999)1n()1n()n(

2

−−

= .

lMhat´TI59 eKman 444355566666,443556666,435666 222 === . 4444355556666662 =

BI«TahrN_xagelIcUrrkrUbmnþTUeTAnigRsaybBa¢ak´rUbmnþen¼pg

dMeNa¼Rsay tambMrab´eKman

444355566666,443556666,435666 222 === 4444355556666662 = .

- 112 - PaK1

Page 120: HU NSEN 3 STR O N G · Five Hundred Mathematical Challenges,Complex Numbers From A to … Z , Mathematical Olympiad in China,Mathematical Olympiad Challenges, Mathematical Olympiad

KNitviTüaCMuvijBiPBelak

- 113 - PaK1

tamlMnaMen¼eyIgGacbeg;ItrUbmnþTUeTAdUcxageRkam ½ 6555.....5553444.....444666.....666

)1n()1n()n(

2

−−

= .

karRsaybBa¢ak´rUbmnþ ½ eyIgtag ½

6555.........5553444..........444A)1n()1n( −−

=

)n(

2

2

)n(

2

)n(

2nnn2

nn2

nn1nn2

1nn1n1n

)1n(

n1n

)1n(

666............666)999.........999(32

)999.........999(94)110(

94

9410.810.4

94)52740(1010.4

9545010.510.2710.410.4

610)110(9510.310)110(

94

610555.......55510.310444........444

=⎥⎥⎦

⎢⎢⎣

⎡=

=−=+−

=

+−−−=

+−++−=

+−++−=

+×++×=

+

−+−

+

dUcen¼eKánrUbmnþ½ 6555.....5553444.....444666.....666

)1n()1n()n(

2

−−

=

Page 121: HU NSEN 3 STR O N G · Five Hundred Mathematical Challenges,Complex Numbers From A to … Z , Mathematical Olympiad in China,Mathematical Olympiad Challenges, Mathematical Olympiad

KNitviTüaCMuvijBiPBelak

- 114 - PaK1

lMhat´TI60 eKman

111111445556,11114556,1156 22 2222 =−=− .

BI«TahrN_xagelIcUrrkrUbm nigRsaybBa¢ak´rUbmnþen¼pg

=−

1111111144455556 22 =−

nþTUeTA

dMeNa¼Rsay rkrUbmnþTUeTA nigRsaybBa¢ak´ ½

22

2

=−

=−

1111111144455556 =− tamlMnaMEdleK[eyIgGacsresrrUbmnþTUeTAdUcxageRkam ½

)n(

nigRsaybBa¢ak´ ½

11111144555611114556

11

22

2

=−

=−

1111111144455556 22 =− tamlMnaMEdleK[eyIgGacsresrrUbmnþTUeTAdUcxageRkam ½

)n2()n()n(

eKman eKman

11111144555611114556

11562

22 =− 562

22 =−

22

22 111.......111445.....444556....555 =− 22 111.......111445.....444556....555 =−)n2()n(

sRmayepÞógpÞat´rUbmnþen¼ ½

)n(

2445...tag )n(

2n ..444556....555T −=

Page 122: HU NSEN 3 STR O N G · Five Hundred Mathematical Challenges,Complex Numbers From A to … Z , Mathematical Olympiad in China,Mathematical Olympiad Challenges, Mathematical Olympiad

KNitviTüaCMuvijBiPBelak

- 115 - PaK1

( )

( )9

999.........999

9110)

9110(110

1)110(941)110(

951)110(

941)110(

95

1)110(941)110(

95

)1444......444(1555.....555=

)n2(n2n

n

nnnn

2n

2n

22

=−

=−

+=

⎥⎦⎤

⎢⎣⎡ −−−+−⎥⎦

⎤⎢⎣⎡ +−++−=

⎟⎠⎞

⎜⎝⎛ +−−⎟

⎠⎞

⎜⎝⎛ +−=

+−+

Bit .

dUcen¼ )n2()n(

2

)n(

111.......111445.....5....555 =

)n2(n 111..........111T =

2 44456 − .

lMhat´TI61 eK[ 011nn aa..........aaA = − nig 011nn a2a.........aaB ×−= −

fa Eckdac´nwg t Eckdacnwg . Rsay lu¼RtaE

A 7 B 7

dMeNa¼Rsay ´nwg ena¼naM[man ebI B Eckdac 7 INq∈ Edl q7B =

eKán q7a2a........aa 011nn =×−− .. naM[ )1(a a2q7a..........a 011nn ×+=−

Page 123: HU NSEN 3 STR O N G · Five Hundred Mathematical Challenges,Complex Numbers From A to … Z , Mathematical Olympiad in China,Mathematical Olympiad Challenges, Mathematical Olympiad

KNitviTüaCMuvijBiPBelak

- 116 - PaK1

eyIgman )2(a10a...........aaaa.........aaA 1nn= − 011nn01 +×= −

yk CUskñúg eKán q7

)1( )2(

00 a)a2(A 10+×+= )a3q10(7a21q07aa20q70A 0000 +=+=++=

naM[ Eckdacnwg . ¼RtaEt Eckdac´nwg .

A 7

dUcen¼ Eckdacnwg luA 7 B 7

lMhat´TI62 eK[cMnYn 0121nn aaa......aaA −=

a.....,,a, Caelx . yfacMnYn Eck alNa

Edl 0 a,a n21

cUrRsa dac´nwg 6 kA

0n21 a)a..aa(4y . ++

dMeNa¼

++= Eckdacnwg 6 .

Rsay eyIgman 0121nn aaa......aaA = −

n2

10 a10.....a10a10aA 2n++++=

Page 124: HU NSEN 3 STR O N G · Five Hundred Mathematical Challenges,Complex Numbers From A to … Z , Mathematical Olympiad in China,Mathematical Olympiad Challenges, Mathematical Olympiad

KNitviTüaCMuvijBiPBelak

- 117 - PaK1

eyIgman )6(mod410 ≡

eyIgán )6(mod410

)6(mod410)6(mod410

n

3

2

−−−−−−−−−−≡

)6(mod)a.....aa(4aA n210 ++++≡ Eckdac´nwg lu¼RtaEt

n1

edIm,I[ A 6

(mod0)a....aa(4a 20 )6≡++++ cen¼cMnYn Eckdacnwg kalNa

2

dU A 6

n1 a)a...aa(4y 0++++= Eckdacnwg 6 . lMhat´TI63 cUrkMntelx a nig edIm,I[cMnYn b abba CaKUbéncMnYnKt´ .

dMeNa¼

Rsay nig kMnt´elx a b

tag ab10b100a1000abbaN +++== b110a1001

+=+=

NN

)b10a91(11

Page 125: HU NSEN 3 STR O N G · Five Hundred Mathematical Challenges,Complex Numbers From A to … Z , Mathematical Olympiad in China,Mathematical Olympiad Challenges, Mathematical Olympiad

KNitviTüaCMuvijBiPBelak

- 118 - PaK1

edIm,I[ N CaKUbéncMnYnKt´lu¼RtaEt *INk,k121k1110a91 332b ∈==+

eday 9b0,9a0 ≤≤≤< ena¼ 909b10a910 ≤+< eKán 909k1210 3 ≤<

smmUl121627 +

121909k0 3 =≤< naM[ 1k =

eKTaján 121b10a91 =+ naM[ 10

a9112b 1−=

ena¼ eday 0b ≥ 0≥10

a91121− ¦

91301a +≤

91121

= naM[ 1a = ehIy 310

91121b =−

= . en¼ dUc nig 3b,1a == 3111331abba == .

Page 126: HU NSEN 3 STR O N G · Five Hundred Mathematical Challenges,Complex Numbers From A to … Z , Mathematical Olympiad in China,Mathematical Olympiad Challenges, Mathematical Olympiad

KNitviTüaCMuvijBiPBelak

- 119 - PaK1

lMhat´TI64 eKtag r nig R erogKñaCakaMrgVg´carikkñúg nigcarikeRkA énRtIekaN ABCmYy . k¿cUrRsayfa

Rr1CcosBcosAs +co +=+

.

x¿cUrRsayfa r2R ≥

dMeNa¼Rsay k¿Rsayfa

Rr1CcosBcosAcos +=++

2BBcosAcos Ccos

2CBcos2

2Asin21Ccos 2 −++−=

++

2Csin

2Bsin

2Asin41

)2

CBcos2

CBcos(2Asin21

)2

CBcos2Asin(

2Asin21

2CBcos

2Asin2

2Asin21 2

+=

−−

+−=

−−−=

−+−=

edayeKmanbc

)cp)(bp(2Asin −−=

ab)bp)(ap(

2Bsin =

p(2Csin,

ac)cp)(a −−

=−−

Page 127: HU NSEN 3 STR O N G · Five Hundred Mathematical Challenges,Complex Numbers From A to … Z , Mathematical Olympiad in China,Mathematical Olympiad Challenges, Mathematical Olympiad

KNitviTüaCMuvijBiPBelak

- 120 - PaK1

eKán abc

)cp)(bp)(ap(2Csin

2Bsin

2Asin −−−

= eKTaj½

abc)cp)(bp)(ap(41CcosBcosAcos −−−

+=+

+

Rr1

R.pr.p1

R.pS1

R.SpS1

RR4

abc.p

)cp)(bp)(ap(p1

2

+=+=+

+=

×

=

−−−+=

dUcen¼ Rr

1CcosBcosAcos +=++ . fa

x¿Rsay r2R ≥

tamvismPaB :GMAM − βα≥β+α .2 eKán )bp)(ap(2)bp()ap( ≥− −−−+

)bp)(ap(2c

)cp)(ap(2bap2

−−≥

−−≥−−

eKTaj )1(21

c)bp)(ap( −≤

dUcKñaEdr )2(21

a)cp)(bp(≤

−−

Page 128: HU NSEN 3 STR O N G · Five Hundred Mathematical Challenges,Complex Numbers From A to … Z , Mathematical Olympiad in China,Mathematical Olympiad Challenges, Mathematical Olympiad

KNitviTüaCMuvijBiPBelak

- 121 - PaK1

nig )3(21

b)cp)(ap(≤

−− ´TMng Gg wgGg:eKán ½ KuNTMnak :n)3(,)2(,)1(

81

abc)cp)(b≤

−−

eKTaj

p)(ap( −

81

2Csin

2Bsin

2Asin ≤

eday 2C

sin2B

sin2A

sinco 41CcosBcosAs +=++ eKán

23)

81(41CcosBcosAcos =+≤++

Et Rr1CcosBcosAcos +=++

aj eKT 23

Rr1 ≤+ ¦ .

r2R ≥

Page 129: HU NSEN 3 STR O N G · Five Hundred Mathematical Challenges,Complex Numbers From A to … Z , Mathematical Olympiad in China,Mathematical Olympiad Challenges, Mathematical Olympiad

KNitviTüaCMuvijBiPBelak

- 122 - PaK1

lMhat´TI65 k¿cUrRsayfa x2cotxcot

x2sin1

−= x¿cUrKNna

n2

n

2aS

2a

sin

1....

sin

1

2a

sin

1asin

1++++=

dMeNa¼Rsay k¿Rsayfa x2cotxcot

x2si

tag n1

−= x2cotxcot)x(f − =

x2sin1

xcosxsin21xcos2xcos2

xcosxsin21xcos2

xsinxcos

x2sinx2cos

xs

=

in

22

2

=+−

=

−−

¼

xcos=

dUcen x2cotxcotx2sin

1−= .

x¿KNna n22

asin

Sn

2a

sin

1....

1

2a

sin

1asin

1++++=

eyIgán ∑=

=n

0kk

n )

2a

sin

1(S eday x2cotxcot

x2sin1

−=

acot

2a

cot

2a

Sn

kkn ⎟⎠⎞=

=∑ cot

2a

cot

1n

01k

−=

⎜⎝⎛ −

+

+

dUcen¼ acot2a

cotS 1nn −= + .

Page 130: HU NSEN 3 STR O N G · Five Hundred Mathematical Challenges,Complex Numbers From A to … Z , Mathematical Olympiad in China,Mathematical Olympiad Challenges, Mathematical Olympiad

KNitviTüaCMuvijBiPBelak

- 123 - PaK1

l atTI6Mh 6

k¿cUrRsayfa xcot2x

cot

xcos1

1 =+ KuN x¿KNnapl

)

2a

cos

11).....(

2a

cos

11)(

2a

cos

11)(

acos1

1(Pn +=

n

+++ 2

dMeNa¼Rsay

k¿Rsayfa xcot2x

cot

xcos1

1 =+ eyIgtag

xcos1

1)x(A +=

xcot2x

cotxtan

2xco

== tanxcosxsin

.

2x

sin

2x

s

2x

sinxcos

xsin2x

cos

2x

sinxcos

2x

sin2x

cos2

xcos2x

cos2

x1x

22

=

===+

=

coscos

dUcen¼ xcot2x

cot

xcos1

1 =+ . x¿KNnaplKuN )

2a

cos

11).....(

22

acos

11)(

2a

cos

11)(

acos1

1(P

n

n ++++=

acot.2

atanP = 1nn + .

Page 131: HU NSEN 3 STR O N G · Five Hundred Mathematical Challenges,Complex Numbers From A to … Z , Mathematical Olympiad in China,Mathematical Olympiad Challenges, Mathematical Olympiad

KNitviTüaCMuvijBiPBelak

- 124 - PaK1

lMhat´TI67 eK[sIVúténcMnYnBit n kMnt´elI eday ½ )U( IN

22

U0 = nig INn,211− U

U2

n1n ∈∀

−=+

aGnuKmn_én . KNna nU C

n

dMeNa¼Rsay KNna GnuKmn_én ½

nU Ca n

eyIgman4π

sin22

0 == U

sin2

sin11

2U11

U1

−=

22

0 =−−

=−

«bmafavaBitdltYTI p KW 2pp s=2π

inU + ayfavaBitdl YTI eyIgnwgRs ´t )1p( + KW 3p1p 2

πsinU ++ = Bit

an eyIgm2

U11U

2p

1p

−−=+ Ettamkar«bma 2pp 2

πsinU +=

eyIgán 2

sin11U 1p+

−−= 2

π2p

2+

3p22 +

dUcen¼

32p sin22π

cos1 +− p2

πsin2

π

2

+=== Bit

2nn 2π

sinU += .

Page 132: HU NSEN 3 STR O N G · Five Hundred Mathematical Challenges,Complex Numbers From A to … Z , Mathematical Olympiad in China,Mathematical Olympiad Challenges, Mathematical Olympiad

KNitviTüaCMuvijBiPBelak

- 125 - PaK1

lMhat´TI68 cUrKNna

9π7

cos9π4

cos9π

cosS 333 +−=

dMeNa¼Rsay KNna

9π7

cos9π4

cos9π

cS = os 333 +− tamrUbmnþ acos3acos4a3cos 3 −= ¦ a3cos

41

acos43

acos3 += GacsresrCa ½ kenßamEdl[

)3π7

cos3π4

s(cos43

S = co3π

(cos41

)9π7

cos9π4

cos9π

+−++− tag

9π7

cos9π4

cos9π

cosM = +− y eda

9π13

cos9π4

cos =−

9π13

cos9π7

cos9π

cosM ++= KuNnwg 3π

sin2 eKán

0)9π7

cos(πsi23

M2 n29π16

sin9π2

sin

9π10

sin9π16

sin9π4

9π10

sin)9π2

sin(9π4

sin3

sin9π13

cos23π

sin9π7

cos23π

sin9π

cos23π

sinM2

=−=+=

−+−+−−=

++=

eKTaján

sin2

M.2

0M =

Page 133: HU NSEN 3 STR O N G · Five Hundred Mathematical Challenges,Complex Numbers From A to … Z , Mathematical Olympiad in China,Mathematical Olympiad Challenges, Mathematical Olympiad

KNitviTüaCMuvijBiPBelak

- 126 - PaK1

tag 23

21

21

21

3π7

cos3π4

cos3π

cosN =++=+−= eKán

83

N41

M43

S =+= .

dUcen¼ 83

97cos

94cos

9cosS 333 =

π+

π−

π= .

lMhat´TI69 eK[RtIekaN mYymanmMukñúgCamMuRsYc .

ayfa Btan.AtanCtanBtanAtan

ABC

k¿cUrRs tan. C=++ ¿TajbBa¢ak´fa x 33CtanBtanAtan ≥++ .

dMeNa¼Rsay k¿Rsayfa tan Ctan.Btan.AtanCtanBtanA =++

= ¦ eyIgman BA ++ πC CπBA −=+ án eK )Cπtan()BAtan( −=+

CtanBtanAtanCtanBtanAtan

CtanBtanBtan

Atan1Atan

+−=

−=

tan.AtanBtanA

+−

+

dUcen¼ .BCtantan Ctan=++ .

Page 134: HU NSEN 3 STR O N G · Five Hundred Mathematical Challenges,Complex Numbers From A to … Z , Mathematical Olympiad in China,Mathematical Olympiad Challenges, Mathematical Olympiad

KNitviTüaCMuvijBiPBelak

- 127 - PaK1

x¿TajbBa¢ak´fa 33CtanBtanAtan ≥++ CamMuRsYc ( tamsm μtikm μ )

eKTaj PaBkUsIueyIgGacsresr ½

eday C,B,A

0Ctan,0Btan,0Atan >>>

tamvism3 Ctan.Btan.Atan3CtanAtan ≥+

eday tan.Btan.AtanCtanBan

Btan + tAtan C=++

eKán 3 CtanBtat nAtan3BtanBtanAan ++≥++

2 ≥++

+ (tan27)CtanBtanA(tan 3

27)CtanBtanA(tan

)CtanBtanA +≥++ dUcen¼ 33CtanBtanAtan ≥++ .

Page 135: HU NSEN 3 STR O N G · Five Hundred Mathematical Challenges,Complex Numbers From A to … Z , Mathematical Olympiad in China,Mathematical Olympiad Challenges, Mathematical Olympiad

KNitviTüaCMuvijBiPBelak

- 128 - PaK1

lMhat´TI70 KNnaplbUk ½ n 999....999....999999S ++++= manelx cMnYn elx ) . ( 9 n

dMeNa¼Rsay KNnaplbUk ½

9.. 99......999. .999999Sn

)9n( elx++++=

9)10n9(10

n9

110.10n

−−

=

)1...111()10...10)110(....)110()110()110(

1n

n32

n32

+−=

++++−+++

−++−+−+−=

+

dUcen¼

1010( +=

9)10n9(10S

2n

n+−

=+

.

Page 136: HU NSEN 3 STR O N G · Five Hundred Mathematical Challenges,Complex Numbers From A to … Z , Mathematical Olympiad in China,Mathematical Olympiad Challenges, Mathematical Olympiad

KNitviTüaCMuvijBiPBelak

- 129 - PaK1

lMhat´TI71 cUrkM ntRKb´KUtémøKt´viC¢man ebIeKdwgfacMnYn ½

)b,a(

1bab2a

32

2

+− CacMnYnKt´viC¢manEdr .

dMeNa¼Rsay nt´RKb´KUtémøKt´viC¢man ½ k

kM )b,a(

y k1bab2

a32

2

=+−

Edl *INk∈ 32eKán 2a2 )1(0)1b(kakb =−+

42

DIsRKImINg´énsmIkar )1b(k4 3bk4 −−=Δ 22 k4)bkb2( 2b−+−=Δ mIkar mancemøIykñúg lu¼RtaEt s )1( *IN Δ CakaerRákd

mann&yfa 2222 dbk4)bkb2( =−+−=Δ

d

Edl CacMnYnKt´. -ebI 0bk4 2 =− ¦

4 bk

2

=

eyIgTTYlán 2

bbb2

kb2a3

2 −=−= ¦

2ba =

Page 137: HU NSEN 3 STR O N G · Five Hundred Mathematical Challenges,Complex Numbers From A to … Z , Mathematical Olympiad in China,Mathematical Olympiad Challenges, Mathematical Olympiad

KNitviTüaCMuvijBiPBelak

- 130 - PaK1

eday a b, CacMnYnKt´viC¢man ehtuen¼eKRtUv[ ½ INp,p2b ∈∀=

eKTaj pp8p4

ehIy

)p2( 42

−=− )p2(2a 2=

pp2== .

2a

2,p(;)p2,pp4dUcen¼ 8()b, *INp,)pa( ∈∀−=

-ebI 0b

k4 2 >−

eKán 4)2 bkb2( 22222 *INk,)1bkb2(dbk ∈∀+−≥=−+− 2¦ )1b(k4 2 0)1b( ≤−+− án eKTaj

IsmI1b =

kñúgkrN kar )1( køayCa 0ak2a2 =− naM[ k2a = dUcen¼ )1,k2()b,a( = cMeBa¼RKb´ *INk∈ . -ebI 0bk4 − 2 <

)bkb −eKán k42( 222222 )1bkb2(db −<=−+− )bkb2 2222 <smmUl 4( 2 0)1bkb2(bk −−−−+−

¦ k4(b2 0)1k4()1b(b2)3 <−+−+− ( minBitkñúg ) srubmkeKánKUcemøIybImanragdUcxageRkam ½

*IN

)k2,kk8(;)k2,k(;)1,k2()b,a( 4 −= *INk∈

Page 138: HU NSEN 3 STR O N G · Five Hundred Mathematical Challenges,Complex Numbers From A to … Z , Mathematical Olympiad in China,Mathematical Olympiad Challenges, Mathematical Olympiad

KNitviTüaCMuvijBiPBelak

- 131 - PaK1

´TIlMhat 72 cUrbgHajfa ½ 1

ab8ca 22

cca8bbc8 2

≥ba

++

++

+

RKb´cMnYnBit ¢man . cMeBa¼ viC c,b,a

dMeNa¼Rsay

ajfa bgH 1ab8

c≥

cca8bbc8aba

222 ++

++

+

CadMbUgeyIgRtUvRsayfa

34

34

34

34

aa2

cbabc8a ++≥

+

mmUl

vismPaBen¼s )bc8a(a)cba( 234

234

34

34

+≥++ smmUl bca8cb2ca2ba2cb 3 3

434

34

34

34

34

34

38

≥8

+++ amvismPaB eKman

+

t GMAM − 34

34

38

38

cb2cb ≥+ eKTaj bca8cb2ba2cb 3333 ca2 3

434

34

34

344488

≥++++ Bit

tuen¼ eh ( )1cba

abc8a

a

34

34

34

34

2

++≥

+

Page 139: HU NSEN 3 STR O N G · Five Hundred Mathematical Challenges,Complex Numbers From A to … Z , Mathematical Olympiad in China,Mathematical Olympiad Challenges, Mathematical Olympiad

KNitviTüaCMuvijBiPBelak

- 132 - PaK1

dUcKñaEdreKTaj ( )2cba

bac8b

b

34

34

34

34

2

++≥

+

ehIy ( )3cba

cab8c2

c

34

34

34

34

++≥

edaybUkvismPaB Gg:nigGg:eKán +

)3(,)2(,)1(

1ab8c

cca8b

bab8a c 222

≥+

++

. ++

lMhat´TI73 eK[ a( n )CasIVúténcMnYnBitEdl

21a1 =

b´cMnYnKt´viC¢man eyIgman nigcMeBa¼RK n1aa

aan

2n

2n

1 =+n +−

cUrRsaybBa¢ak´facMeBa¼RKb´cMnYnKtviC¢man n eyIgman ½ n3 aa 21 1a.........a <+++ .

dMeNa¼+

Rsay RsaybBa¢ak´fa 1a.........aaa n321 <++++ eyIgman

1aaaa 1n =+

n2

n

2n

+−

Page 140: HU NSEN 3 STR O N G · Five Hundred Mathematical Challenges,Complex Numbers From A to … Z , Mathematical Olympiad in China,Mathematical Olympiad Challenges, Mathematical Olympiad

KNitviTüaCMuvijBiPBelak

- 133 - PaK1

tag n

n a1b = ena¼TMnak´TMngEdl[køayeTACa ½

n2 +−=

bb1 −=−

1bbb n1n+

eKTaj 1nb + n2

n

nnnn1n 1b)b(b1b−

−=

−=

nn1 )1b(b1n

b11

111

b −=−

+

+

eKTaj 1b1bb

a1nnn

n −−

−==

+

111

n21n21 .aa ++

b1......

b1

b1a... +++=+

11b

111b1b 1n1 − +

111b1b 121 −−− +

)1b

11b

1(...)1b

11b

1()

1n

nn32

<−

−=−

−=

−−

++−

−−

+

+

BIeRBa¼

11( −=

2a1b

1

dUcen¼ 1a.........aa

1 == .

a n321 <++++ .

Page 141: HU NSEN 3 STR O N G · Five Hundred Mathematical Challenges,Complex Numbers From A to … Z , Mathematical Olympiad in China,Mathematical Olympiad Challenges, Mathematical Olympiad

KNitviTüaCMuvijBiPBelak

- 134 - PaK1

lMhat´TI74 cUrkMntRKb´KUtémøKt´ 3n,m ≥ ebIeKdwgfacMeBa¼RKb´ cMnYnKtviC¢man eKman a

1aa1aa

2n

m

−+−+ CacMnYnKt´ .

dMeNa¼Rsay

´KUtémøKt´viC ½ kMnt´RKb ¢man ( )n,m

edIm,I[ 1aa

an1aa2n

m

−+−+ CacMnYnKt´lu¼RtaEt 2 −1a+

aktþarYmén C 1aam −+ ehIy . Igyk

nm >

ey *INk,knm ∈+= Igán ½

k1k2nk

knm

−+−+−+

−+=−++

+

,I[

ey

)1aa)(a1()1aa(a1aa1aa

=

tamTMnak´TMngen¼edIm 1aa 2n −+ CaktþarYmén a 1am −+

lu¼RtaEt 1kn += n

ig 2k = . .

dUcen¼ )3,5()n,m( =

Page 142: HU NSEN 3 STR O N G · Five Hundred Mathematical Challenges,Complex Numbers From A to … Z , Mathematical Olympiad in China,Mathematical Olympiad Challenges, Mathematical Olympiad

KNitviTüaCMuvijBiPBelak

- 135 - PaK1

lMhat´TI75 eK[bIcMnYnKt c,b,a Edl ba´viC¢man = 10c++ . cUrrktémøFMbMputén cbaP ××= . dMeNa¼Rsay rktémøFMbMputén cbaP ××=

aM[ ceyIgman 10cba =++ n ba01 −−= )b10(b10(ab 2 −eyIgán baP aa)b +−=−−=

2tag )ba(P ab10() 2ba −+−= eKán 2bb10ab2)a('P −+−= bI eKTaján e 0)a('P =

2b5a −=

eday a(''P *INb,0b2) ∈∀<−= øGtibrma mø ehtuen¼ )a(P mantém cMeBa¼té

2b5a −= .

a¼ RtUvEt YnKU I

eday *INa∈ en CacMnb

eb k2b = ena¼ k5a −= ehIy k5k2)k5(10c −=−−− =

Page 143: HU NSEN 3 STR O N G · Five Hundred Mathematical Challenges,Complex Numbers From A to … Z , Mathematical Olympiad in China,Mathematical Olympiad Challenges, Mathematical Olympiad

KNitviTüaCMuvijBiPBelak

- 136 - PaK1

eday ,a ena¼ ⎪⎩

⎨*INc,b ∈ ⎪⎧

≥−=≥=≥−=

1k5c1k2b

1k5a ¦ 4k1 ≤≤

= n

-cMeBa¼ k 1

eKá 4c,2b,4a === naM[ 32424P =××=

n 3a

-cMeBa¼ 2k =

eKá c,4b, 3== P= naM[ =× 36343×= -cMeBa¼ 3k = eKán 2c,6b,2a == naM[ 242P= 26 =××= -cMeBa¼ 4k =

n a eKá 1b,1 c,8 === naM[ 32181P =××= témøGtibrmaén dUcen¼ c.b.aP = es wg .

μIn 36

Page 144: HU NSEN 3 STR O N G · Five Hundred Mathematical Challenges,Complex Numbers From A to … Z , Mathematical Olympiad in China,Mathematical Olympiad Challenges, Mathematical Olympiad

KNitviTüaCMuvijBiPBelak

- 137 - PaK1

lMhat´TI76 eKmansmPaB

ba1

bxcos

axin 44s

+=+

0Edl b,a 0ba,0 ≠+≠≠ a cUrbgHajf 33

8

3

8

)bxcos

axsin

=+ .

eNa¼

ba(1+

dM Rsay gHajfa b 33

8

3

8

)ba(1

bxcos

axsin

+=+

eKman ba

1b

xcosa

xsin 44

+=+

M[ na abxcos)ba(axsin)ba(b 44 =+++

0)xsinbxcosa(

0xsinbxsinxcosab2xcosa)xcosx(sinabxcosabxcosaxsinbxsinab

222

422242

422442424

=−

=+−

+=+++

eKTaj ba

1ba

xa

xsin=

cosxsinb

xcos 2222

+++

==

naM[ 44

8

4

8

)ba(1

bxcos

axsin

+==

Page 145: HU NSEN 3 STR O N G · Five Hundred Mathematical Challenges,Complex Numbers From A to … Z , Mathematical Olympiad in China,Mathematical Olympiad Challenges, Mathematical Olympiad

KNitviTüaCMuvijBiPBelak

- 138 - PaK1

eKTaj )1()ba(

aa

xsin43

8

+= nig )2(

)ba(b

bxcos

43

8

+=

nig Gg:nigGg: eKán ½ bUksmIkar )1( )2(

343

8

3

8 cxsin+

)ba(1

)ba(ba

bxos

a +=

++

=

dUcen¼ 33

8

3

8

)ba(1

bxcos

axsin

+=+ .

lMhat´TI77 eK[RtIekaN ABC mYYy . bgHajfaebI

3Ctan,

3Btan,

3Atan

Ikar bxaxx:)E( 23Ca¦ssm c 0=+++ n ena¼eKá cb3a3 +=+ .

dMeNa¼Rsay karbgHaj ½ eyIgman π=++ CBA

MukñúgRtIekaN ) yIgán ( plbUkm ABC

e ]3C)

3B

3Atan[()

3C

3B

3Atan( ++=++

Page 146: HU NSEN 3 STR O N G · Five Hundred Mathematical Challenges,Complex Numbers From A to … Z , Mathematical Olympiad in China,Mathematical Olympiad Challenges, Mathematical Olympiad

KNitviTüaCMuvijBiPBelak

- 139 - PaK1

)1()

3Ctan

3Btan

3anta

3tan

3(tan1 +−

Ct3

nBA3Ctan

3Btan

3AtanCta

3Btan

3

3Ctan.

3Btan

3Atan

3Btan

3Atan

3Ctan

3Btan

3A

3BtanA

3Ctan

31

3Ctan)

3B

3Atan(

+

+

+

++

++=

A3

ntan3

−=

A+

11

−−

tan1

3

tan

3tan =

π

)3BAtan(

)3

tan(+

CBA ++

eday 3Ctan,

3Btan,

3Atan Ca¦srbs´smIkar

ena¼tamRTwsþIbTEvüteKmanTMnak´TMng ½ )E(

)2(a3Ctan

3Btan

3Atan −=++

)4(c2Ctan.

2Btan.

2Atan

)3(b2Ctan

2Atan

2Ctan

2Btan

2Btan

2Atan

−=

=++

ykTMnak´TMng nig CYskñúgsmIkar eKán ½ )3(,)2( )4( )1(

Page 147: HU NSEN 3 STR O N G · Five Hundred Mathematical Challenges,Complex Numbers From A to … Z , Mathematical Olympiad in China,Mathematical Olympiad Challenges, Mathematical Olympiad

KNitviTüaCMuvijBiPBelak

- 140 - PaK1

b1ca3

−+−

= ¦ cab33 +−=− dUcen¼ cb3a3 +=+ .

lMhat´TI78 k¿ cUrKNnatémø®ákdén

10sin π nig

10cos π

x¿ cUrRsayfa 10

sin)yx(4)yx(x 22222 π+≤−+

RKb´cMnYn IRy,x ∈ .

dMeNa¼Rsay k¿ KNnatémø®ákdén

10sin π nig

10cos π

eKman 1032

210π

−= eKán

ππ

103cos)

103

2sin(

102sin π

−π

tamrUbmnþRtIekaNmaRt ½ nig acosasin2a2sin = acos3acos4a3cos 3 −=

Page 148: HU NSEN 3 STR O N G · Five Hundred Mathematical Challenges,Complex Numbers From A to … Z , Mathematical Olympiad in China,Mathematical Olympiad Challenges, Mathematical Olympiad

KNitviTüaCMuvijBiPBelak

- 141 - PaK1

)1010

sin1(43sin2

10cos43

10sin2

10cos4

10cos

10cos

10sin2

103cosco

10sin2

2

3

π−−=

π

π−=

π

π−

π=

ππ 3

10s

2

π=

ππ

¦ 0sin2sin4 2 =11010

−π

−π tag 0

10 eKán t4 2

sint >π

>

=

0541',01t2 =+=Δ=−− ¦s eKTaj 0

451−t1 <= ( minyk )

451t, 2

+=

dUcen¼ 4

5110

sin +=

π . eday 1

10cossin 22

10=

π+

π

naM[ 4

5210)4

51(110

2 −=

+−=

π cos

dUcen¼ 410

cos =5210 −π .

sayfa x¿ R10

sin)yx(4)2 ≤yx(x 2222 π+−+

tagGnuKmn_ 10

sin)yx(4)yx(x)y;x(f 22222 π+−−+=

Page 149: HU NSEN 3 STR O N G · Five Hundred Mathematical Challenges,Complex Numbers From A to … Z , Mathematical Olympiad in China,Mathematical Olympiad Challenges, Mathematical Olympiad

KNitviTüaCMuvijBiPBelak

- 142 - PaK1

eKán ½ 222222 )

451)(yx(4yxy2xx)y;x(f +

+−+−+=

IRy,x,0y2

15x2

15

y2

15xy2x2

15

y5 2

21xy2x

251

253)yx(yxy2x2

1626)yx(4yxy2x2

22

2

2222

2222

∈∀≤⎟⎟⎠

⎜⎜⎝

−−

⎟⎟⎠

⎞⎜⎜⎛ +

++−

+−−

−=

++−+−=

++−+−

5=

2⎞⎛ +

+=

=

dUcen¼ 10

sin)yx(4)yx(x 22222 π+≤−+ .

Page 150: HU NSEN 3 STR O N G · Five Hundred Mathematical Challenges,Complex Numbers From A to … Z , Mathematical Olympiad in China,Mathematical Olympiad Challenges, Mathematical Olympiad

KNitviTüaCMuvijBiPBelak

- 143 - PaK1

lMhat´TI79 eK[sIVúténcMnYnBit kMnt´eday)U( n 4

nsin.2Un

= Edl . k-cUrbgðajfa

*INn∈

4nsin

4ncos

4)1n(cos.2 π

−π

=π+

x-Taj[ánfa 4

)1n(cos)2(4

ncos)2(U 1nnn

π+−

π= +

K-KNnaplbUk ½ n321n U.........UUUS ++++= CaGnuKmn_én .

dMeNa¼

n

Rsay a k-cUrbgHajf

4nsin

4ncos

4)1n(cos.2 π

−π

=π+

tamrUbmnþ bsinasinbcosacos)bacos( −=+ Igán ½ ey

)4

sin4

nsini4

cos4

n(cos2

)44

ncos(24

)1n(cos2

ππ−

ππ=

π+

π=

π+

4

nsin4

ncos)4

nsin22

4ncos

22(2

4)1n(cos2 π

−π

−π

=π+

Page 151: HU NSEN 3 STR O N G · Five Hundred Mathematical Challenges,Complex Numbers From A to … Z , Mathematical Olympiad in China,Mathematical Olympiad Challenges, Mathematical Olympiad

KNitviTüaCMuvijBiPBelak

- 144 - PaK1

dUcen¼ 4

nsin4

ncos4

)1n(cos.2 π−

π=

π+ .

x-Taj[ánfa 4

)1n(cos)2(4

( 1nn

ncos)2U n π+−

π= +

an eyIgm4

nsin4

ncos4

)1n(osc.2 π−

π=

π+ naM[

4)1n(cos2

4ncos

4nsin π+

−π

KuNGg:TaMgBIrnwg n)2( eKán

4)1n(cos)2(

4ncos)2(

41nnnsin)2( n π+

−π

= +

Ucen¼

π

d4

)1n( π+cos)2(4

ncos)2(U 1nn −nπ

= + . K-KNnaplbUk n321n U.........UUUS ++++=

eyIgán

4

)1n(cos)2(4

cos2

4)1 πk(cos)2(

4kcos)2(

1n

n

1k

1kk

π+−

π=

⎥⎦⎤

⎢⎣⎡ +

−Snπ

=

+

=

+∑

dUcen¼ 4

)1n(cos)2(1S 1nn

π+−= + .

Page 152: HU NSEN 3 STR O N G · Five Hundred Mathematical Challenges,Complex Numbers From A to … Z , Mathematical Olympiad in China,Mathematical Olympiad Challenges, Mathematical Olympiad

KNitviTüaCMuvijBiPBelak

- 145 - PaK1

lMhat´TI80 eK[ n cMnYnBit 0a.......;;a;a;a n321 ≥ . eKtag *NSn In,

na.....aaa n321 ∈∀

++++=

yfaebI

cUrRsa 1n1nan3211n a.......a.a.aS +

++ ≥ ena¼eKán n

3n .....S ≥

n21 aaaa .

dMeNa¼Rsay ebI Rsayfa 1n

1nn3211n aa.......a.a.aS +++ ≥

ena¼eKán nn321n

eyIgman a.....aaaS ≥

1n1nn3211n aa.......a.a.aS ≥ +

++ eday

na.....aaa

S n321n

++++ =

ena¼ 1n

aa....aaaS 1n

1nn321

++++++

= + eKán ½

+

( )1aa.........aaa 21 aaa

1naa..... 1n

1nn3211nn3 +

++ ≥

++++

k

++

yn

a.....aaaa n321

1n

++++=+ CYskñúg ( )1 eKán ½

Page 153: HU NSEN 3 STR O N G · Five Hundred Mathematical Challenges,Complex Numbers From A to … Z , Mathematical Olympiad in China,Mathematical Olympiad Challenges, Mathematical Olympiad

KNitviTüaCMuvijBiPBelak

- 146 - PaK1

nn21

n21

n21 a.a≥n

n2

n21n21

1nn21

1n n221

n2

1n n21n21

n21n

a.........a.an

a.......aa

a......n

a.......a

na...aa

a.....a.an

a.........aan

a....a....a.a

na.......a

na....aa

.a...a.ana.....aa

≥+++

⎟⎠⎞

⎜⎝

+++

++≥⎟

⎠⎞

⎜⎛ +++

+++≥

++

+++≥

1a⎛

1n

1 a.a

a1n+

+

21 a....aa

...+

++++

+

+

+

+++

eKTaj nn321n

dUcen¼ ebI a.....aaa S ≥

1n1nn3211n aa.......a.a.aS +

++ ≥ ena¼eKán n

n321n a.....aaaS ≥ .

lMhat´TI81 cUrbgHajfa

101

10099......

65.

43.

21

151

<<

dMeNa¼Rsay bgHajfa

101

10099......

65.

43.

21

151

<< an eyIgm )1n(n)1n( +>+ 2n1n2:*INn +=+∈∀

eKTaj 1n

n2n21n2

+>

++

Page 154: HU NSEN 3 STR O N G · Five Hundred Mathematical Challenges,Complex Numbers From A to … Z , Mathematical Olympiad in China,Mathematical Olympiad Challenges, Mathematical Olympiad

KNitviTüaCMuvijBiPBelak

- 147 - PaK1

eKán

251

5049......

32.

21

10099......

65.

43

=>

KuNnwg 21 eKán

2101

10099........

65.

43.

21

>

eday 151

2101

>

eKTaj )1(151

10099....

65.

43.

21

> müa¨geTot

)1n2)(1:*INn −∈∀ n2(2)1n2()1n2(n4 +>++−= eKTaj

1n21n2

1n2n2

−+

>−

rW 1n21n2

n2n2 − 1

+−

<

eKán 1011

10199....

75.

53.

31

10099.....

65.

43.

21

=<

eday 101

1001

1011

=<

eKTaj )2(101

10099....

65.

43.

21

< Mng nwg eKán ½tamTMnak´T )1( )2(

101

10099......

65.

43.

21

151< < .

Page 155: HU NSEN 3 STR O N G · Five Hundred Mathematical Challenges,Complex Numbers From A to … Z , Mathematical Olympiad in China,Mathematical Olympiad Challenges, Mathematical Olympiad

KNitviTüaCMuvijBiPBelak

- 148 - PaK1

lMhat´TI82 eK[ n2 ....; cMnYnBit ;b;b;a.....;;a;a .

a....b 2n

22

21

2n

22

21

2n22 ++++++≤++

n21n21 b

cUrRsayfa ½ )b.....bb)(a....aa()baba( n11 +

dMeNa¼Rsay

.....bb)(a....aa()ba....baba( 222

21

2n

22

21

2nn2211 +++++≤+++

eyIgeRCIserIsGnuKmn_mYykMnt´

Rsayfa ½ )bn+

IRx∈∀ eday ½ )bx..)bxa()b

2n

221nn2211

22n

22

2

2n

222

21

+++++++++

+++++

eday )b...bb(x)ba..baba(2x)a...aa()x(f 21 +=

a(...xa()x(f n1 +=

IRx∈∀ RtIFa 0)x(f ≥ Canic©ena¼

eday KánCanic©

⎩⎨⎧

≤Δ>

0'0af

0a.....aaa 2n

22

21f >+++=

ehtuen¼e 0'≤Δ . ( ) 0)b....bb)(a....aa(ba. ...baba' 2

22

1n211 ++++++=Δ .... 2n

22

21

2n

2n2 ≤+++−

dUcen¼ a()ba....baba( 2n

22

21

2n

2nn2211 ++++≤+++ )b.....bb)(a....a 2

22

1 ++

Page 156: HU NSEN 3 STR O N G · Five Hundred Mathematical Challenges,Complex Numbers From A to … Z , Mathematical Olympiad in China,Mathematical Olympiad Challenges, Mathematical Olympiad

KNitviTüaCMuvijBiPBelak

- 149 - PaK1

lMhat´TI83 eK[ )( nig y( nxn CasIVútcMnYnBitkMnt´elI

0 = nigTMnak´TMngkMenIn ½ n1n yx3xx +=+ nig RKb´

cUrKNna nig CaGnuKmn_én .

dMeNa¼

) IN

eday ,5x0 =3

1y2

nn3

nn2

n1n yyx3y +=+ INn∈

n

x ny n

Rsay

eKman nnn1n +=+ nig

Ikar eKá)yx(y

n

3nn1n

+=+

+=+

ln{ n

KNna nx nig ny CaGnuKmn_én23

n

)1(yx3xx

)2(yyx3y 3nn

2n1n +=+

bUksm nig 2( n ½ )1( )

)yxln(.3)yxln(x 1n ++

n1n1n ++

TMnak´TMngen¼bBa¢ak´fa x( n })y+ CasIVútFrNImaRtman ersug nigtYdMbUg 3q = 6ln)yxln( 00 =+ .

eKán 6ln3)yxln( nnn =+

Page 157: HU NSEN 3 STR O N G · Five Hundred Mathematical Challenges,Complex Numbers From A to … Z , Mathematical Olympiad in China,Mathematical Olympiad Challenges, Mathematical Olympiad

KNitviTüaCMuvijBiPBelak

- 150 - PaK1

eKTaj nig eKán ½

.3yy 3

n

−+

¢ak´fa

)3(6yxn3

nn =+

dkksmIkar yx =−

)1( )2(

)yx()xln()x(

nn1nn

nn11n

−=++

+ ln1

TMnak´TMngen¼bBa })yxln({ nn − CasIVútFrNImaRtman ersug nigtYd 003q = MbUg ln( 4ln)yx =− .

Taj nig eKán

eKán 4ln3)yxln( nn− n =

eK )4(4yxn3

nn =−

bUksmIkar 3( ) )4(nn 33

n 46x2 +=

eKTaj 2

46xnn 33

= . n+

ar nig eKán −= dksmIk y2)3( )4(nn 33

n 46

eKTaj 2

yn46

nn 33 −= .

dUcen¼ 2

46xnn 33 +

n = nig 2

46ynn 33

n−

= .

( anPaKTI2 sUmG )

sUmrgcaMG rKuN

Page 158: HU NSEN 3 STR O N G · Five Hundred Mathematical Challenges,Complex Numbers From A to … Z , Mathematical Olympiad in China,Mathematical Olympiad Challenges, Mathematical Olympiad

KNitviTüaCMuvijBiPBelak

- 151 - PaK1

Éksareyag ( Referent )

Hu hematical Chal es dward J. Bar beau, M ray S. Klamkin)

plex Numbers Fro A to … Z itu Andresscu , Dorin Andrica )

a Oly in China at ly hallenges

itu Andresscu & n Gelca) ematical Oly d Treasures

ndreescu , Bogdan enescu ) 7. International Mathematical Olympiads 1959-1977

8. The IMO Compendium ( 1959-2004 )

(By Titu Andreescu , Dorin Andrica )

1. 103 Trigonometry Problems (From the training of the USA IMO Team)

2. Five ndred Mat leng (By E ur

3. Com m (By T

4. Mathem tical mpiad 5. Mathem ical O mpiad C (By T Razva

6. Math mpia (By Titu A

( Samuel L. Greitzer )

9. 360 Problems for Mathematical Contest

Page 159: HU NSEN 3 STR O N G · Five Hundred Mathematical Challenges,Complex Numbers From A to … Z , Mathematical Olympiad in China,Mathematical Olympiad Challenges, Mathematical Olympiad

KNitviTüaCMuvijBiPBelak

- 152 - PaK1